What about subjective experience implies anything non-physical?

Mary’s room is a classic philosophical thought experiment about consciousness.  The Wikipedia article on what’s called the knowledge argument quotes Frank Jackson, the originator of the argument, as follows:

Mary is a brilliant scientist who is, for whatever reason, forced to investigate the world from a black and white room via a black and white television monitor. She specializes in the neurophysiology of vision and acquires, let us suppose, all the physical information there is to obtain about what goes on when we see ripe tomatoes, or the sky, and use terms like ‘red’, ‘blue’, and so on. She discovers, for example, just which wavelength combinations from the sky stimulate the retina, and exactly how this produces via the central nervous system the contraction of the vocal cords and expulsion of air from the lungs that results in the uttering of the sentence ‘The sky is blue’. […] What will happen when Mary is released from her black and white room or is given a color television monitor? Will she learn anything or not?

The takeaway idea from this thought experiment is supposed to be that, since Mary knows “all the physical information there is to obtain” about seeing color, what she learns when having her first actual sensory experience of color must be non-physical.

But this assumes that it is possible for Mary to actually know everything physical about seeing color, without actually ever seeing color.  It seems clear she does get new knowledge when she leaves the room, the knowledge of what it’s like to actually experience color.  The question is what the nature of that new knowledge is.  Like so many of these types of exercises, the premise essentially assumes the conclusion, that raw subjective experience isn’t physical.  But if the raw experience actually is physical, then the premise is a contradiction, positing that she has all the information, then going on to describe what information she doesn’t have.

But the question I have is, why does this premise, that experience is not physical, seem compelling to so many people?  (At a philosophical level.  I understand why so many people find it emotionally compelling.)

One of the chief features that separate humans from other animals is the degree to which we can think symbolically.  Language is the most common example of this ability.  Other animals issue sounds which mean something to those around them, such as a monkey who issues a certain screech for a snake, and a different screech for a flying predator.  But only humans appear able to manipulate the sounds in complex sentences and frameworks, particularly with hierarchical and recursive levels of complexity.

When we use language, we utter a sound that is a symbol for something else.  That something else might be another symbol acting as another placeholder for collections of more primitive symbols.  But eventually, if we follow through the hierarchy of symbols, the most primitive ones we can find will represent sensory perceptions, emotions, or actions, in other words, raw conscious experience.

Now, you might argue that some words refer to objects, such as dogs.  But dogs are themselves a composite sensory experience.  When I say the word “dog” to you, it evokes certain imagery.  But the dog concept generally denotes a certain type of animal with a certain type of body plan.  The imagery has colors, textures, shapes, sounds, and smells, in other words, more primitive sensory experiences.

We might also talk about the altered consciousness of meditative states some people experience.  But if you read descriptions of those states, they’re always either using a new word to label that state, or attempting to describe it in terms of the other primitives we’re all familiar with.

So, all language ultimately reduces to these primitive aspects of conscious experience: sensory perception, primal emotions, motor action, and perhaps meditative states.  Once we reach this point however, language ends.  While we can come up with words as stand-ins for these primitives, we can’t further describe them.

For example, consider trying to describe the color yellow to someone who had been born blind.  You can’t.  The best you can do is attempt to relate it into terms the blind person might understand, such as the feel of sunshine, the touch and smell of bananas, etc.  But you can’t describe the raw experience of yellow to them.  It’s ineffable.

But does this ineffability, this inability to subjectively reduce the raw experience further, mean anything about the reality of such an experience?  What about this ineffability might lead us to conclude it involves something other than physics?

It’s worth noting that just because these experiences can’t be subjectively reduced, it doesn’t mean that the neural correlates can’t be objectively reduced.  For example, we know the experience of yellow begins with photons with wavelengths of between 575 and 585 nanometers striking our retina, exciting a mixture of red sensitive and green sensitive light cone receptors and causing a cascade of electrochemical signals up the optic nerve to the thalamus and occipital lobe, somewhere producing what will eventually be communicated as yellow to the other brain centers.

Of course, we are far from a full accounting of the neuroscience here.  And many seem always ready to seize on the remaining gaps as an opportunity to wedge in mystical or magical notions.  But every year, those gaps close a little more.  Taking solace in them seems like an ever eroding stance.

A common argument is that we don’t know why these experiences exist.  Why can’t the brain go about its business without them?  This seems to assume that raw experience is superfluous to what the brain does, and perhaps that superfluousness means that it’s outside of the causal framework we call “physics”, an epiphenomenon.

But as I’ve noted before, the very fact that we can discuss primal experiences and apply symbolic labels to them means that they’re not outside of that causal framework.  It takes extreme logical contortions to avoid concluding they don’t influence at least the language centers of our brain.

So then, what explains experience?  As I’ve noted before, I think to have any hope of answering that question, we have to be willing to ask what experience actually is.  It seems like there are many possible answers, but the one I like best is grounded in the evolutionary reason for brains, to make movement decisions.  Experience is communication.  But communication from what to what?

I think the answer is: communication from the perception centers and emotion centers of the brain to the movement planning centers.  This communication provides information that is crucial for the movement planning centers to do their job.  What we call “experience” or “feeling” is the raw substance of that communication.  This communication includes sensory perceptions (including a sense of self) and emotional reactions.  Remove it, and it’s difficult to see how movement decisions can happen.

Of course, this remains a speculative explanation.  Any explanation of experience will be at this point.  The question is, does speculation of this type, built on physical functionality we already know has to exist in the brain, involve fewer assumptions than speculation about non-physical phenomena?

It’s often said that subjective experience can’t be explained physically.  My question is, what am I missing?  What about experience causes people to say this?  What specific attributes are outside the purview of any such explanation?

99 thoughts on “What about subjective experience implies anything non-physical?

  1. Unfortunately for you , your explanation will never satisfy more then a small group of people. That our configuration of atoms and molecules produces our raw experiences, granted we can get into. But the intensity of suffering and pain, the heaviness of a depression, the amazingness of the drug XTC(mdma) or magic mushrooms. Nobody is satisfied that the reason for the incredible intensity of these feelings is for movement planning. That is until science proves this beyond any reasonable doubt. till then 99,999% of all people will not be satisfied with your best explanation.
    Sorry. Nice post again.

    Liked by 1 person

    1. Thanks for being honest about why you think people reject it. 99.999% is an exaggeration (most scientists see the mind as completely physical), but I agree that most people simply don’t want it to be true. It’s the answer I expected, although I’m open to hearing something more substantive.

      Like

  2. Wonderfully said. It evoked the scene in The Wizard of Oz movie in which it transitioned from a black and white movie to one of full color. The desire for magic to be real is real, why I do not claim to know. I once thought that people didn’t want reality to be reducible to “cold, hard facts” because they were inept at learning facts and would be excluded from fundamental knowledge, but that, like much of the rest of thinking surrounding this issue, is a wild ass guess.

    It seems that the whole purpose of imagination is as a tool for survival. If I can anticipate the future I can act accordingly, and where could these anticipations be acted out? Only in a mental space devoted to imagining. Just because we have this ability, it is not limited to just survival-based functions, especially when the Homeland Security survival threat code drops to green, creating lots of idle time for such a powerful function.

    That our sensory apparatus was designed to be able to secure information from far to near and input it into a processing capability is clear and does not preclude that apparatus being captured by hallucinogens, etc. Heck, I can press on my eyeballs and get some pretty weird visual patterns. And so what? None of this points to supernatural or “spiritual” phenomena unless we want them to. So, the question is why do we want ghosts and spirits in our lives? As protectors, sources of evil, a form of special knowledge of which only a few are aware of that makes us “special”? (Thank you, Church Lady!)

    Liked by 1 person

    1. Thanks. I suspect most people want it to be true because they’d rather be ghosts themselves after death than non-existent. That, and a world filled with supernatural wonders is seen as more exciting than one filled with the scientific view of matter and energy.

      Like

      1. Mike,

        Does physical extend into other dimensions? Can there be any multi-dimensional experiences that affect us? Perhaps our true nature is better understood as a metaphysical manifestation. Can anyone point to their ‘being’, or existence? Yet no one would deny having the experience of being and existing. We just keep trying to relate it to something that we can grasp with our mind. It seems to me that existence is much more than we comprehend and extends multidimensionally. The problem arises when we think of ourselves as a person with a narrative within the scope of time. Your being never changes. You are the same being as you were when you were a kid, it’s just the forms that change and the various experiences we have that come and go and linger in memory. Once you know this, experience this, the filter of subjectivity gets punctured to allow this knowing to show you your own nature. Then you begin to feel connected to everything in a wonderful way. Are you with me, Mike? lol

        Liked by 2 people

  3. Great article as always. I really like how you explained this thought experiment and the nice little dig about thought experiments usually begging the question. So true!

    You’re right; the issue comes down to whether we really know all the facts of the matter and we don’t. Language represents sensory information we already have, and it’s this sensory information that’s the real information, not the language itself.

    Chalk one more up to the bait and switch from the misuse of language 🙂

    Liked by 2 people

  4. This reminds me of the idea that textbook learning is never quite the same as hands-on experience. If you’re studying to be an electrician, for example, you won’t really know how to do the job until you actually go and do it. But I doubt that wiring an electrical outlet for the first time is some grand, mystical experience.

    Liked by 2 people

    1. Good example.

      It’s also occurred to me before that the act of writing is itself something you don’t know how to do until you’ve done a certain amount of it. And the flow of creativity is something we can’t control, at least not as consistently as we’d like. It’s probably not a coincidence that the ancients anthropomorphized that somewhat capricious creative flow into a muse.

      Liked by 2 people

  5. Great post Mike.

    I think the reason that so many are compelled by non physical explanations, is because that’s exactly how what we perceive seems when we don’t understand how something works, or “magical.” The rise of science in recent centuries has done a great deal to remove the shroud of mystery, but of course most of us are pretty ignorant about a great many things. I know I am. The difference between us versus those who consider there to be more than causality, I think, is that they haven’t taken this model and carried it through to everything else. Hell I’m such a freak with causality, that I don’t even accept what most physicists accept, or that Heisenberg’s uncertainty principle mandates an ontologically uncertain future. They may be right about that, though to me their interpretation of this principle implies the existence of magic.

    Liked by 2 people

    1. Thanks Eric.

      It’s interesting that most people don’t really understand how computers work, but they’re fine accepting that their internals are fully materialistic. They even worry that at some point those computers might “wake up” and become a threat to humanity, but never wonder if there’s a non-physical aspect to their operation.

      But the idea that human minds might be nothing more than electrochemical signalling, essentially a wet computational system, is repugnant to many. Of course, the difference is that one is the other, and one is us. It reminds me of Descartes consigning non-human animals to automaton status, but assuming that there just had to be something more to humans.

      I’m personally not dogmatic about causality. I think quantum physics obliges us to be open minded about it. But I also have limited patience for people who reach for quantum explanations only because they dislike the implications of deterministic neural information processing.

      Liked by 2 people

  6. Fully agree. I’ve been contemplating something like this for the last week, since I saw this comment come up in my feed, where it was assumed as a given that “not objectively describable” equates to “non-physical”? Why? My guess is that people are falling victim to the fallacy of denying the antecedent. Everything known through objective means is physical and so the converse must also be true – that everything known by non-objective (subjective) means is therefore not physical.

    Liked by 2 people

    1. Hmmm. Hadn’t seen that comment, or blog before. Thanks!

      What inspired my post were statements from people as diverse as David Chalmers, Philip Goff, and neuroscientist Christof Koch, that it’s necessary to posit consciousness as some extra fundamental force to reality, similar to electromagnetism or the nuclear forces.

      Excellent point on the denying the antecedent fallacy.

      Liked by 1 person

  7. Do you actually think that Mary when she comes out of the room, and is confronted with all kind of colors, actually can point out the color red. So when she comes out of the room she is tested and asked from several colored papers to pick up the red one. To me it is not obvious she can. I think somebody really has to teach her which color is which. And what does this imply? I don’t know just some extra thoughts.

    Liked by 2 people

    1. If she’s confronted with color papers, I think you’re right, that she would need someone to teach her what each color was. If she sees objects, such as a rose, she could probably pick out the color by context, although she might reach the wrong conclusion if it’s an off-color version of the object.

      But there’s actually a much starker issue that would likely arise if anyone ever tried this in the real world. Knowing what we do about nervous system development, there’s a good chance she may be incapable of distinguishing colors. The neural circuitry typically recruited to make those determinations might have gotten recruited for other tasks. Or her ability to do so might be severely impaired.

      There’s a period in vision development known as the critical period, where if an infant has a vision impairment, such as large cataracts on their lens, and it’s not corrected within the first few years of life, they may never have fully functional vision. In a real life version of the thought experiment, Mary may never be able to distinguish colors as well as someone raised seeing them. (As an expert in neuroscience, she would presumably predict this consequence.)

      Liked by 1 person

      1. You are right about that. Also I once started a book from an indian neuroscientist, Can’t reproduce the name really. It was about synesthesia and a color blind person. So when he saw certain shapes he saw strange colors his color blind eyes could not perceive in the real world. He called them colors from mars.
        I just red Sean Carrol’s the big picture. He also had something to say about Mary’s thought experiment. He said that mary’s neurons who perceive red had never fired of course in the black and white room. So that is something physical. On the one hand I really like philosophical thought experiments, on the other I wonder if they can produce real insight. What does it mean to know everything about red. etc. I think for scientific process they are quite worthless actually.
        I remember reading the thought experiment and thinking about a virgin philosopher who read everything about sex and all there was to know. When he left his room without women do we think he would gain any new experience by having it.
        Also we could maybe say that if adding a certain amount of information doesn’t bring us any closer to understanding red, like the wave length etc. We could say that also all the information would not bring us any closer to understanding it.

        Liked by 2 people

        1. Was that Indian neuroscientist Vilayanur Ramachandran? He’s done some interesting TED talks and a number of people have praised his books.

          Sean Carroll’s ‘The Big Picture’ has been on my reading list for a while. (I’ve just been more attracted to neuroscience books lately.) I think he’s right that Mary’s lack of experience amounts to the related neurons never having been fired. Interestingly, all the color sensitive light cones on her retina would have been triggered when she saw white, since white is technically a combination of all those colors, but the circuits of neurons in her brain that make the distinctions never would, and would likely have been recruited for other tasks.

          Thought experiments can be useful for science. Einstein was reportedly able to formulate special and general relativity using them. Of course, those theories weren’t generally accepted until they started passing actual experimental tests.

          Einstein’s thought experiments were obviously much higher quality than a lot of the philosophical ones that get discussed. The problem with thought experiments is their quality is in the eye of the beholder, with no objective test to settle arguments. I know many people quite taken with Mary’s Room, The Chinese Room, and other similar exercises, but those people already agreed with, or were predisposed to agree with the conclusion.

          Good point about the virgin philosopher. That’s one we can probably all relate to. Many of us learn a lot about sex before actually having it, but I think just about everyone would admit we learn from the act itself.

          Like

        1. Thanks Tina! That is interesting. However, along with the ophthalmologist, I’m skeptical the glasses are actually allowing him to see red. The fact that the manufacturer wouldn’t provide a pair for scientific evaluation is telling. I hope his parents didn’t pay too much.

          I suspect what the glasses are actually doing is shifting or compressing the color spectrum to bring red into his visual range as a shade of the colors that he can see. That is useful. He would be able to discriminate colors he couldn’t before. Never having actually seen red, he wouldn’t know the difference. But it doesn’t seem likely to get him into the air force.

          Liked by 1 person

          1. Funny, I saw this on the news shortly after reading your blog post and I normally wouldn’t have paid much attention, since, who cares about the feel-good story. So you think when he sees a pink shirt, he’s still seeing brown, or in any case, not really pink? How sad. I believe the kid bought those glasses with his own money. I won’t tell if you won’t. 😉

            Liked by 1 person

        2. There might be more to this than I thought. Turns out in most cases of color blindness, the person does have all three types of cones. What actually causes their color blindness is that the sensitivity ranges between their different types of cones overlap too much, preventing them from being able to distinguish between some colors. The glasses appear to work by filtering out the overlapping wavelengths, actually giving the person the ability to distinguish colors they couldn’t before.
          https://www.technologyreview.com/s/601782/how-enchromas-glasses-correct-color-blindness/

          Liked by 1 person

  8. Mike, help me out with this, would you? To define something as physical, necessarily connotes its opposite, doesn’t it? If it doesn’t, then why even bother with physical definitions? Is it sufficient to define this opposing quality of non-physicality purely in such negative terms — i.e. the absence of the physical? Well, even if it is, we still have ‘something’ set apart from the physical; we have an existent quality. Is that existent quality magic?

    Liked by 2 people

    1. Hariod, as we’ve discussed before, defining “physical” can be problematic. But I think for purposes of this discussion, we can define it as that which operates according to the laws of physics, more specifically, the laws of chemistry and electricity, as well as the paradigms of biology.

      I’m not quite sure if I understand what you mean by “existent”. Are you using it in a way distinct from “existence”? What about it makes it non-physical?

      Liked by 1 person

      1. Thanks Mike. I think if you look at what I said just from a logic point of view, without abstracting it and narrowing it into physics and biology, then ‘something’ stands in contradistinction to the physical. That non-physical ‘something’ must be an existent (as a quality in itself) otherwise the concept of ‘physical’ loses meaning. So, is this ‘something’, that is non-physical, just a magical conception, an illusion, or whatever?

        When you say, “What about it makes it non-physical?”, then that might imply that you think everything is physical; so again, why the need to define any phenomenon whatsoever as ‘physical’ if there is nothing other than the physical?

        By ‘existent’ I mean to suggest there is some referent quality to the term, even if only expressed in the negative, such as ‘non-physical’.

        Liked by 1 person

        1. Hariod, I appreciate the clarification. If “physical” is that which operates according to the laws of physics, then I think the non-physical would necessarily be that which violates those laws. Of course, our understanding of those laws are imperfect, but again for purposes of this discussion, I think we could stipulate the currently well established physical laws.

          But are you saying that because we can describe something like that, that it implies its existence? If so, this seems close to the ontological argument, that something must exist by virtue of its definition. The problem, of course, is we can also imagine all kinds of things: elves, dragons, mermaids, and unicorns, which the overwhelming majority of us agree don’t exist, at least not outside of the neural firing patterns that make up our imagination.

          Why the need to refer to the non-physical? Well, I’m pretty sure I wouldn’t be if there weren’t so many people insisting that it’s necessary to explain our mental life. So maybe using the definition above, my question would be: what about subjective experience implies violations of the currently understood laws of physics?

          Liked by 1 person

          1. Thanks Mike. Oddly enough, I think we’re at the same point because you also are saying that ‘physics’ is a meaningless term, in effect, in that you only use it as a contradistinction to the idea that there may be ‘something’ (howsoever conceived) beyond the observable and measurable universe. For yourself (you will tell me if this is incorrect), there is nothing other than the (potentially) observable and (potentially) measurable universe.

            You also are saying that the Laws of Physics are provisional, which I understand, and presumably therefore can and will be adapted to accommodate phenomena that may yet be discovered but which would transgress the current laws. If that is so, then again it makes a nonsense of defining phenomena as physical, which itself may sound a little like the Ontological Argument — we’ve simply switched the meaningless concept of ‘God’ for the equally meaningless concept of ‘Physicality’.

            I know I’m being tedious in labouring this point, but in coming back to our common sphere of interest — that of the mind and consciousness, The Mind/Body Problem, etc. — then to me it seems we may be couching the whole debate within a meaningless paradigm given that the goal posts for the physical universe are movable to include anything and everything. Is that so different to the position of the Transcendental Idealist (again, not me) who says everything, science included, necessarily reduces to consciousness?

            Liked by 1 person

    2. Hariod,
      Actually, I’m open minded about there being things that we may never be able to observe or measure, even indirectly. But from an epistemic view, they might as well not exist for us since we have no way to assess them, and we have no good reason to think any speculation we have about them has the remotest chance of being right.

      On the physical vs non-physical, I understand the point you’re trying to make, and I don’t necessarily disagree. I’ve often wondered if the distinction was coherent. I think it comes down to whether there are phenomena that don’t follow some kind of regularities we usually call physical laws.

      But on the mind/body problem in particular, I don’t think this is a meaningless question, because it goes to whether the mind can be scientifically understood. If someone a priori believes that it is beyond scientific understanding, then I can understand whether it is ultimately physical or not is irrelevant. But if it is composed of well understood physics, then it’s something amenable to scientific investigation.

      Therefore, the reasons why people think it can’t be explained that way are important, at least to me. I personally can’t see any logical reason to assume the mind is beyond scientific exploration. But maybe I’m missing something, hence this post.

      Liked by 1 person

      1. Thanks once again for indulging me, Mike. Still, I don’t think I’ve shifted ground on the matter of physicality being a somewhat artificial distinction as a result of this exchange. Taking what you just said, “I’ve often wondered if the distinction was coherent. I think it comes down to whether there are phenomena that don’t follow some kind of regularities we usually call physical laws.”, then again I come back to the fact that were that to be so, and that these phenomena did not follow whatever the current Laws of Physics may be at the time of their discovery, then the laws simply get changed so as to co-opt these new phenomena into a so-called ‘Physics’. So really, to say that everything known is physical, is as meaningless as saying everything only ever exists as consciousness. But anyhow, thanks once again for indulging me, and I hope you don’t think I’ve been unnecessarily contrarian over this point.

        Liked by 1 person

        1. Hariod, I always enjoy our discussions, so my pleasure. Again, I understand and don’t necessarily disagree with the point you’re making.

          Maybe for purposes of this discussion, you could view it as a question of whether there is anything in subjective experience that implies that our understanding of fundamental natural laws needs revision.

          Liked by 1 person

          1. Well, isn’t it fair to say that we don’t actually have any ‘fundamental natural laws’ as regards subjective experience? All we seem to have are an array of quite widely varying schemas for what happens in the brain when it arises. It seems to me that the Laws of Physics need firstly to account for awareness, or knowingness, rather than asking what about the laws (as they stand) need revising in order to account for it. I guess it depends on whether one takes awareness as an a posteriori given, a self-evidenced general principle — i.e. not a reasoned out proposition. Intuitively, and rightly or wrongly (no one knows), one takes it to be the former. Then, as we try to reason out its existence, the intuitive ‘certainty’ is lost.

            Liked by 1 person

  9. Mike,

    You still haven’t addressed what I asked you in my above post regarding whether the physical extends into other dimensions. It would be hard to imagine there not being other dimensions that do not affect our physical bodies. Oddly enough, what everyone refers to matter of factly as existence has no locus or physical center, yet no one will deny that they exist and so does the universe. Let’s subtract the universe and ourselves and what is left is space. Is space not physical? If not, what is it?

    Liked by 2 people

    1. Jeff, my apologies. Somehow I completely missed your comment yesterday.

      I don’t think I’d agree that we have the same being from when we were a child. We have the same genetics, but I doubt most people would regard that as our being. It seems like our being is a creation of both our nature and experiences. But if so, then every experience we have alters us, so that we’re not the same from experience to experience, from year to year. I’ll grant that our earliest experiences have an out sized effect compared to later ones, but a sufficiently moving or traumatic experience as an adult seems able to change change us in profound ways.

      Does the physical extend into other dimensions? I don’t know. Certainly if you buy into string, brane, or multiverse theories, then they might. But the lack of unique evidence for any of those theories keeps me skeptically agnostic.

      The thing is, even if the physical does extend in that way, is there any reason to suspect it wouldn’t do so with the same types of regularities (i.e. laws) that we observe with our current dimensions? We have to be careful of looking just beyond the edge of the map of knowledge and writing “Here be dragons”, since most of the time, after the map got extended, there weren’t anything as exotic as dragons there. (And usually when there were, such as quantum mechanics or black holes, they were unexpected.)

      Liked by 1 person

      1. Mike,

        There is nothing wrong with what you’ve put forth, but it seems to be incomplete, if I may use that word. When I said your being doesn’t change, what I am referring to is not your physical body or what you think or feel about anything. It is the nature of all of this, its essence, so to speak, which is your basic state. You can call it awareness as Hariod does, or anything else. But, having the experience of your own nature, introduces a knowingness that is not based on the physical, mental, or any other phenomenal appearance which is related to our current dimensional existence. It is not a thing and not a state of mind, but it embraces all of it and reveals the same nature that all appearances have. That knowingness is essential. It is what we are, but we are not attentive to that, we are attentive to our own models of self, subjectivity, and relentless craving. All of this gets resolved in this knowingness. Including the dragons………:-)

        Liked by 1 person

        1. Jeff,
          I suspect what you’re describing is something that many people intuitively feel is there, an unchanging core of self. It seems like there are two possibilities:
          1. That this unchanging core of self, this knowingness, exists as an addition to everything else we can talk about: perceptions, emotions, memories, imagination, a bodily sense of self, etc.
          2. That our brain has a data representation in it of this concept, not because it exists in and of itself, but because it’s a useful simplified stand-in to represent the entire organism, around which to anchor survival instincts and related primal impulses.

          The question is, how can we tell the difference? It doesn’t seem possible to do so subjectively, since if 2 is true, the very introspection mechanism, the inner eye we use to perceive this irreducible core can’t see past the stand-in.

          It seems like the only option is to look for objective evidence for 1, that is, evidence independent of our subjective perceptions. If 2 is true, then we shouldn’t find any such evidence. Of course, many will insist that absence of evidence is not evidence of absence. But how long do we hold out the possibility of 1 being true in the face of that absence?

          Like

          1. Mike,

            Yours are astute observations and analyses. As I said, this experience of your innate nature, is not a core nor a thing, but resolves the questions posed by subjectivity, your mind that thinks in terms of time and all its experiences it has accumulated that give you a sense of core and self. It is no longer trying to find out who or what it is as an answer to questions that create the subjective self. It is as if another dimension has been introduced into your view. Out of this knowingness, all experience is no longer happening to you, it is just happening, like the clouds in the sky. It is all of the same stuff and it is all integrated into this knowingness. It has to be experienced, but our own questioning of everything stands in the way. It happens in an instant, when you really see that there is no solid you at the center of experience. In fact, none of these questions have anything to do with what you are, they are all learned through cultural adaptation which is conditioned and cannot lead to this knowingness. The mentation doesn’t disappear completely, but quiets down considerably and loses its force of habit because you don’t engage it anymore. It is no longer your primary tool of life. Only through the force of contemplation, can you comprehend this. I know this goes against what you believe or think you perceive, but belief is ultimately useless and not the point. It’s a leap, but throughout history there have been those who have made that leap. Perhaps the word metaphysics takes on a different flavor.

            Liked by 1 person

  10. Hariod,
    I think you’ve made some fine observations above. If I define how to get on the bus, then this also defines how not to get on the bus, given that that must then be everything else. But what happens if someone happens to be on the bus that never got on? Well surely my “get on the bus” definition was crap!

    For defining the “physical” or “natural,” I like to skip epistemological notions and go directly to the ontological concept of “causality.” This is why I’m so disturbed that modern physicists dumped causality and threw Einstein under the bus, simply to have their expansive interpretation of quantum mechanics (ie: “Things are fundamentally random and thus spooky”), work out. Einstein wasn’t this epistemologically arrogant.

    I doubt that we’d have quite as many problems here if people were a bit more epistemologically responsible. But then aren’t philosophers currently in charge of the field of epistemology? And don’t these people still fail to reach any such agreements? Thus I humbly ask for associated professionals to consider the following two principles.

    EP1: There are no true definitions, but only more useful ones in the context of a given argument. (So if someone defines a term, stop saying “You’re wrong”! Just use it in the attempt to understand their point.)

    EP2: There is only one process by which anything conscious, consciously figures anything out. It takes what it thinks it knows (evidence), and uses this to assess what it’s not so sure about (theory). (If what you think you know doesn’t conform with your theory, then you move on, even though what you think you know could actually be the problem.)

    Liked by 1 person

  11. Reading the comments here, sometimes quantum mechanics comes up. From what I understood from Sean Carroll’s book the big picture, even a quantum universe is completely deterministic. There is no place for true libertarianism. The Quantum possibility’s are fixed. Not even free will can change them in the end. An all knowing person could from the beginning on predict all possible states with their according probabilities.
    The book is a good read.

    Liked by 2 people

    1. Carroll favors a certain view of quantum physics, the Everett many worlds interpretation. That does preserve determinism, but at a cost that, to my mind, makes it a Pyrrhic victory. Quantum events are not random, but all possible events happen. Even if that interpretation is the right one, the fact remains that for our personal subjective timeline, quantum events effectively remain random.

      That said, I’m not convinced that quantum physics is needed to understand the mind. It’s warm, wet, and electromagnetically noisy in the brain, not an environment generally seen as favorable for preserving quantum effects. I haven’t seen any evidence to indicate that we need to go any lower than electricity and molecular chemistry to understand what’s happening. Not that this makes things simple by any measure.

      Liked by 1 person

    2. Thanks for the clarification Mike. I was about to respond something like, “Yaay, a notable physicist is on my side!” Models such as “Many Worlds,” however, are far too exotic for me. I read a review of the book that Oscar suggested and it didn’t even mention that little tidbit (though it did coincidentally go into “Mary the color scientist” scenario).

      Liked by 1 person

      1. I probably should have issued the disclaimer that I haven’t read his book. I’m basing my remarks above on what he’s written about it on his blog. But he reiterated that position in a recent interview connected with the book, so it sounds like that’s still where he is.

        Liked by 1 person

        1. Thank you for your clarification .you are always so well informed. You remembered even better, while I read the book. (actually the audio edition) Yes I must say a couple of years ago I decided I couldn’t wrap my mind about something from nothing. So why not a infinite universe. Why not infinitely many?
          Something from nothing is still stranger. Also some sort of shock was that all the energy in the universe added up to zero. We may find out in the future that all the space and all time in the universe also adds up to exactly zero and nothing is there. It’s nothing from nothing and all the universes are ways to add things up to nothing. But this is beyond this article of Mary.

          Liked by 1 person

          1. Thanks!

            It’s weird, but all the energy adding up to zero has never bothered me that much. It might be because my bachelor’s degree is in financial accounting, where all the transactions, all the credits and debits, no matter how big the organization, must add up to zero. (If they don’t, your books are “out of balance”.) It seems appropriate that accounting for energy seems to work the same way.

            Like

        2. I highly recommend you read the book. It’s a fairly easy read and has not much to do with Everettian QM. Also, he would agree with you that classical physics is (almost certainly) sufficient to explain mind/consciousness.

          Liked by 1 person

  12. I would like to take a shot at answering the question: “why does this premise, that experience is not physical, seem compelling to so many people?”.

    I think the answer is that the content of experience is not physical, even though the cause of the experience is physical, and the content of the experience is all we have subjective access to.

    This answer presumes a kind of dualistic ontology. There are two kinds of things that are “real”, namely physical things, and patterns (hey!). Patterns do not “interact” with physical things, but patterns can be discernible in physical things. When a given pattern is discernible in a physical system, that system can be said to be “informed” with that pattern. By my definition, information is a specific (well-defined) pattern embodied (discernible) in a physical system.

    So let’s consider as a physical system an electromagnetic wave of a specific wavelength (red) propagating through whatever. There may be another physical system (cone cell?) “designed” to discern the former system (red light) and generate an appropriate response, i.e., squirt a neurotransmitter. We can say that the cone cell “recognized” the “information” in the red light and generated new information (neurotransmitter) in response. So what, if anything, got passed from the red light to the neurotransmitter? Some might say information, but I say that is an abuse of the concept of information (and is a specific contradiction of how I have defined information). Instead, I would say that semantics got passed from the red light to the neurotransmitter. Strictly speaking, the semantics did not get “passed” because it is not a physical thing. Instead, I would say that the neurotransmitter contains the semantics/meaning of “red” because there is a causal chain to a physical system that embodied the pattern of red.

    Now consider a system whose entire visual input is that one cone cell, such that this system can detect the neurotransmitter. The system in question has a robust auditory system and verbal output, such that you can converse with it. When red light hits that cone cell for the first time, the system has access to that neurotransmitter for the first time. But it doesn’t know anything about the neurotransmitter. It only knows it has access to something new, and the only thing it knows about that new thing is that the new thing “means” red. So the system has access to a non-physical pattern (red) without knowledge of any of the physical things that led to that access.

    *

    Liked by 2 people

    1. James,
      Thank you for giving an answer! You made me do a little research, which is awesome.

      You started with patterns. This is something that, as I’m sure you’d imagine given the name of this blog, I’ve put some thought into. First, I completely agree that the patterns are an important aspect to reality. I’m a reductionist, but I’m not an eliminative reductionist, because I think the patterns very much matter. The thing that separates a biological system from a bag of random organic chemicals are the patterns they’re arranged in.

      You make a distinction between physical things and patterns. But for a pattern to exist, doesn’t it have to be a pattern of something? A book’s existence transcends any one copy of the book, but only because there are other copies in the world. If we burn every copy of that book, does it still exist anymore? Perhaps someone has memorized the book, and so retains the information in their brain. But aren’t the synaptic patterns in their brain related to the book, essentially another copy of it? So if we burn every copy of a book and no one remembers it anymore, does the book still exist?

      But you might insist that while it exists, the pattern of the book’s contents remain separate from any one physical instantiation, and so there remains a distinction. But what is a book, but a pattern of paper? What is paper but a pattern of pulp, of dead tree cells? What are cells but patterns of DNA, RNA, proteins, and lipids? And what are those things but patterns of molecules, which are patterns of atoms, which are patterns of electrons, neutrons, and protons. The neutrons and protons are patterns of quarks. And all of it may be patterns of quantum field excitations, strings, branes, or some other more primitive reality, which may ultimately be patterns of something else. There’s a chance it may be patterns all the way down.

      All of which is to say that the distinction between the physical and patterns may be artificial, a distinction between things that feel solid to us on human scales, and those that don’t.

      And now for the part that you made me do research on 🙂

      “So what, if anything, got passed from the red light to the neurotransmitter?”

      Let’s start with the physics. Photons are packets of electromagnetic energy. When they strike the electron shell of an atom, they increase the energy state of whatever electron they hit. That causes the electron to move to a higher energy shell. Which causes the electron to have to do something. As I understand it, what happens depends on the electron configuration of the atom (and/or molecule) and the energy of the photon.

      The electron may simple emit a new photon, allowing it to fall back to its previous state. In other words, the atom just reflected the light. It will reflect photons with certain amounts of energy, but not others, which is why certain materials have the colors that they do.

      But the combination of photon energy and electron configuration may completely knock the electron loose from its atom or molecule. When this happens on a large enough scale, it causes a flow of electrons, an electrical current.

      This is the physics at the center of a process called visual phototransduction, where photons striking a photoreceptor cell are converted into electrical energy.
      https://en.wikipedia.org/wiki/Visual_phototransduction

      So what gets passed from the red light to the neurotransmitter is energy. Light energy is converted into electrical energy, which causes cascades of electrical and electrochemical signalling, the stuff of the nervous system.

      So it doesn’t seem like there was any opportunity here for anything non-physical to happen. But maybe I missed something?

      Liked by 1 person

      1. Mike,

        You’ll notice that I did not use the word “exist”. I said patterns are “real”, intentionally taking a cue from Dennett’s “Real Patterns”. All of math is patterns. Also, there are some patterns that cannot be “informed” in matter other than by symbolic reference. Infinity is one. Complex numbers is another. Self-contradictory concepts is another (as in, the well-known married bachelor). I believe this puts me in the camp that says chess was real before it was invented. But it didn’t exist until it was informed, instantiated.

        You bring up (essentially Donald Hoffman’s point) that we don’t experience reality because we don’t know all of reality. It’s “patterns all the way down”, but we don’t know if there is a bottom. But several important people are recognizing that we don’t have to go all the way down. Luciano Floridi talks about Levels of Abstraction. Dennett started with stances (physical, design, and intentional), but in his recent book (of which I am about halfway through) he talks about (someone’s concept of) affordances, and refers to (somebody else’s) use of the term “umwelt”. Finally, pretty much the whole point of Sean Carroll’s book is what he calls “poetic naturalism”, which means we have different language to talk about what is “real” depending on what level of the [book -> paper-> -> -> molecule ->-> boson -> -> ?] scale we’re talking about. The point in each case is that to ask a question, you must choose a context (level), and then identify a discrete,finite set (pattern) of measurable data which makes sense at that level. You can then measure those data, and if the measurements are accurate and the data are appropriate (Floridi would say well-formed) for that level, then those measurements provide information about reality. They don’t provide all possible information about that reality, but you don’t need that.

        Regarding neurons and energy, I don’t think there is any meaningful way to say that energy gets transferred from the light to the neurotransmitters. Agreed, the entire process is physical, and requires energy. But as I said, strictly speaking, nothing gets transferred from the one to the other. Transfer is a physical concept of taking something from “here” and depositing that thing “there”.

        Consider a very simple chain of neurons with no branching, so,

        Red light –> cone cell –> neurotransmitter CC –> neuron A –> neurotransmitter A –> neuron B –> neurotransmitter B

        Let’s assume neurotransmitter A is different from neurotransmitter B, say, (for no good reason) GABA and dopamine. The only thing that all of the neurotransmitters have in common in this arrangement is the semantic content of “red”. But here I am using “content” figuratively. What each set of neurotransmitters “has” is an informed pattern and a causal history. In fact, that causal history goes all the way back to at least the Big Bang. But for the cone cell and each neuron in that chain, the pertinent part of that causal history is presumed to be the red light part, because that is where a pertinent difference was discerned.

        I guess you could say that what gets transferred to the neurotransmitter is causal history, thus, a pattern, but one that is necessarily tied to physical events.

        Does that help any?

        *

        Liked by 2 people

        1. James,
          “I believe this puts me in the camp that says chess was real before it was invented.”

          It sounds like you’re a Platonist. I’m somewhat agnostic toward Platonism, seeing it as a possible but not necessary paradigm. My current philosophy of mathematics leans toward semi-empiricism. I think the “unreasonable effectiveness of mathematics” is because we build mathematics based on real world patterns.

          I’m about halfway through Dennett’s book myself right now. I got somewhat bogged down in the memes chapter and decided I needed a break. (Which I’m spending reading sci-fi novels and a book on the prefrontal cortex.)

          I agree that we should definitely use levels of abstraction as a tool for understanding things, but that’s just an epistemic framework. Reality itself doesn’t use or care about those levels. They’re only a tool for us hominids to ponder reality well outside our original ecological niche. And sometimes entities from what we define as a lower level will affect entities on what we perceive at a higher level.

          “Regarding neurons and energy, I don’t think there is any meaningful way to say that energy gets transferred from the light to the neurotransmitters.”

          I have to admit that I’m a bit puzzled by this statement. It seems like you’re asserting that energy can’t cross from the causal entity, the photon, to the effected entity, the electron in the light cone cell, or anywhere else along the causal chain to the neurotransmitters. But if that’s true, then energy never flows across any causal-effect relationship, which seems to deny most of physics, including the kinetic energy one billiard ball transfers to another one when it strikes it, or the transfer of chemical and electrical energy to mechanical energy in a car motor.

          Granted, the transfers of energy with these exchanges are pretty weird when you think carefully about them, but it appears to be a fundamental way in which the universe works.

          Like

          1. Mike,
            I think I’m a small p platonist, although I’m not sure what that would mean in this context.

            Re: levels, you said “[r]eality itself doesn’t use or care about those levels.” I think this statement is exactly wrong. That was Donald Hoffman’s point. Living things care about their environment, and if they sample that environment at the wrong level, they lose competitiveness/fitness.

            Re: causal chain
            My turn to be confused. I’m not saying there is no interaction between the photon and the cone cell. I’m saying there is no physical relation between the initial photon and the final neurotransmitter at the end, except the causal chain. If instead of starting the causal chain with a photon, we start it with an electric shock to neuron A, the pattern of neurotransmitter B will be exactly the same, and it will be interpreted as “red”, mistakenly. Also, we could imagine that neuron A branches out to 1000 neurons B, and each of those branches out 1000 times. If there was some sort of energy transfer from an original photon, would that energy be thus a million times diluted? Instead I’m saying each of those neurotransmitter dumps would have different information (pattern), but they would all share some causal history, and it’s that shared history that is pertinent. They would all be equivalent with respect to their “meaning”.

            *
            [tempted to ask your understanding of how neurons work]

            Liked by 1 person

        2. Hi James,
          I’ve been working pretty hard to comprehend your position, both given that I love the epistemic responsibility that you display, as well as because I see similarities with a position of mine. Unfortunately for me however, I don’t seem sufficiently gifted to get it at this point. But instead of killing myself over this, I’ve decided to explain things to you from my perspective to see if you can relate it over to yours? Here goes…

          Though I’m about as “physicalist” as they come, I do not consider our languages, such as math and English, to be physical. Personally I’m hesitant about calling a statement like “2+2=4” real, but to each their own regarding definition. Regardless, like you I consider this sentence to only exist to the extent that it’s physically expressed. From here I can continue this theme to say that everything conceptual could at least be referenced as something, though without existing. As I see them, “jello” and “5” are simply conceptual terms. Thus your “patterns” seems to include my “language,” though perhaps it contains more.

          I’ll now physically invoke the “2+2=4” sentence. Thus it should exist as such, even though an underlying language or pattern can have no reaction with the physical. So is it your position that many of us decide that because these nonexistent patterns are often interpreted to exist, that there must be something more than the physical? And regardless, do you yourself suspect such notions to be false?

          Like

          1. Eric,

            I didn’t see anything wrong with what you wrote.

            To answer your last question, I do suspect that notions of anything non-physical “existing” are false.

            To answer your penultimate question, I’m not sure I want to commit to that position. Some people definitely do conclude that something non-physical exists and is involved in subjective experience. Quite a few others seem to conclude that everything is physical, but subjective experience involves some physics we don’t know about yet (like quantum effects in microtubules?).

            Dennett’s latest book references a book by C. Eliasmith, “How to build a brain: An architecture for neurobiological cognition”, which apparently discusses “semantic pointers”. [yet another book to read. Hopefully there are papers on the net that describe his position.] I have a feeling that this concept may be related to what I’m talking about. I’m hoping that I will be able to say that subjective experience is just “what it feels like” to have access to a set of semantic pointers.

            *

            Liked by 1 person

        3. James,
          My Platonist / platonist observation wasn’t meant as a direct point in the discussion. Just a friendly observation. There are some platonists who visit here periodically (both of the upper and lower case variety). One is a pretty strong mathematical platonist whose views you might find interesting.

          “Living things care about their environment, and if they sample that environment at the wrong level, they lose competitiveness/fitness.”
          I agree. Our perception of the world, what some call the “manifest image”, is one geared toward making effective movement decisions to maximize the preservation and propagation of our genetic legacy. However, while a wolf cares about catching food and avoiding predators that could harm it, it can be affected by things below the level of its manifest image, such as a virus, or by licking a poisonous chemical.

          Again, I do agree that levels of abstraction are a critical epistemic tool. But we need to remember that it’s only a tool we use to understand things. Like any tool, it won’t always be useful.

          “If there was some sort of energy transfer from an original photon, would that energy be thus a million times diluted?”
          Obviously the photon doesn’t supply the only energy for the whole system. When I push my foot on my car’s accelerator pedal to make the car go, I’m initiating a process that generates far more movement than I can exert with just my foot. Most of the energy comes from the incoming oxygen and the gasoline in the fuel tank. But it seems strange to say the sequence from my foot pushing the pedal to the car reaching 60 mph has a non-physical component.

          Likewise, the photon initiates a sequence, but that’s all it does is initiate it. Most of the energy for action potentials, neural signalling, comes from the gradient across the neural membrane between the ions inside and outside of the neuron. When the action potential reaches the presynaptic membrane, it triggers a process that delivers vesicles of the neurotransmitters to the gated channels and across the synaptic cleft, where they are taken up by the postsynaptic membrane, which triggers more processes that may eventually result in the postsynaptic neuron firing.

          I do agree that all of these components share a causal history. But that seems similar to the causal history between my foot and my car’s wheels. (Admittedly the neural chain is much more complicated.) I know you said above that you didn’t like the word “information” in this context, so I’ve avoided it until now, but information processing seems like a productive description.

          But what I’m still struggling with is understanding where you see something non-physical, at least in a way that is unique from a similar process in a robot’s antenna receiving radio transmissions (electromagnetic radiation) that eventually trigger electrical currents through particular silicon transistors.

          Like

          1. Mike,

            We’re on the same page as to what happens physically, then. Sorry.

            So what I see as non-physical in the chain from photon to neurotransmitter is the association between the photon and the neurotransmitter that is assumed by whatever “sees” only the neurotransmitter. Like I said above, the chain could have been started by an electric shock, but the neurotransmitter would still be associated with “red” by processing that comes after. You should be able to see the difference between this chain and the chain from your foot to the speeding car. There’s nothing about the car that particularly cares about your foot.

            BTW, I don’t dislike the word information. What we’re talking about is very much information processing. It’s just that I have a very strict definition, and most people don’t have my understanding of information. So, by my definition, neurons do not transfer information. Instead, information is recognized, a process happens, and new information is generated. Shannon information, i.e., digitally coded information, is a strict subset of my definition, and this information IS transferred via wire or wireless communication. The “form” is the same at both ends.

            *
            [neurons do transfer semantics … semantic pointers?]

            Liked by 1 person

        4. James,
          It sounds like we’re square on physicalism, which is hopful since our ideas might then end up complementing each other. I’m one of the people you’ve mentioned who theorizes subjective experience to be physical. This is because I know that I exist (via René Descartes’ theory), and specifically given my subjective experience (which I consider as conscious processing , or “thought”). Thus with my belief in physical existence, as well as my knowledge that I must be existing right now, it follows that subjective experience must be physical (that is if it’s true that I do happen to be physical). Any quibbles there?

          It interests me that you’re looking for prominent people with ideas that can be incorporated into yours, such as for your subjective experience theory. I’ve had only slight luck doing so to to this point myself.

          I define subjective experience as input to the conscious processor, and it comes in three varieties. First there is a punishment/reward form (that I’ve lately been calling “affect” around here, though “happiness” works as well). This motivates conscious function. Then there are “senses” that provide further information, as in “touch” and “vision.” Then finally there is subjective experience by means of past conscious experience that has been recorded for the present, or “memory.”

          By the way in my own models, no physical terms, such as “synapse,” are presented. I consider myself more “architect” than “engineer.”

          Like

        1. There are no stupid questions. (Well, there are, but this isn’t one of them. 🙂 )

          No one really knows what energy is, at least aside from vague definitions like “the capacity to perform work”. But matter is concentrated energy, so energy itself is usually considered physical by definition. Moving matter has kinetic energy. And there are all kinds of energy like the potential energy of a rock held above the ground before it is released.

          What we can say for sure is that one type of energy gets transformed into another type all the time. Your car converts electrical and chemical energy into kinetic energy.
          Hydroelectric dams convert the mechanical energy of passing water into electrical energy.

          For purposes of this discussion, the photons striking the light cones get converted into electrical and then electrochemical energy, at least if the photon has the right level of energy (i.e. wavelength or color), otherwise it may simply be bounced back out.

          Like

  13. James,
    Thanks for the clarification. No worries on the actual physics.

    Let me try to reword what you said, and you can tell me if I’m getting it. The experience of red is separate from the photon striking the retina. If someone’s occipital lobe is stimulated in the right way, they will have the experience of red without the photon or retina being involved. The normal relationship between light coming into the eye and our perception of it can be subverted.

    Of course, the way we care about seeing red could itself conceivably be subverted, perhaps if someone stimulated the right part of the upper brainstem, amygdala, or other parts of the limbic system. So the normal relationship between the perception of red and our feeling about it can also be subverted.

    But the key thing maybe, is that the prefrontal cortex wouldn’t know that any of these relationships had been subverted. It would receive information about red being perceived, and receive feelings about that perception.

    Normally there would be a relationship between what comes in and what is perceived, between the inner world and interactions with the external world, what some neurobiologists call an isomorphism. I think this relationship is the main reason for consciousness, to concentrate information about the environment in order to make better movement decisions.

    It sounds like what you’re saying is that this relationship is not physical. If so, I can see a way of looking at it where that’s true. It seems similar to the relationship a self driving car’s internal models has to its environment. But if that’s accurate, then I think I take your point. I’m not sure I’d agree (much depends on whether we think a relationship is meaningful without the actual related items), but I can see the point.

    Like

    1. Mike,

      I agree with everything you said …

      … except … maybe … this:

      “But the key thing maybe, is that the prefrontal cortex wouldn’t know that any of these relationships had been subverted. It would receive information about red being perceived, and receive feelings about that perception.”

      At the risk (hope?) of starting a new tangent, I would change that paragraph to:

      But the key thing maybe, is that the [thalamus/claustrum/basal ganglia] wouldn’t know that any of these relationships had been subverted. It would receive information about red being perceived, and [generate feelings, and then] receive feelings about that perception.

      Don’t know if you wanna go there.

      *

      Liked by 1 person

      1. Hmmm. Well, if we’re going to have that discussion, we should take care to ensure we’re not talking past each other with differing definitions. So there is the generation of an experience, and there is the receiving of it. And there is the generation of an emotional feeling, and there is the perception of it.

        When I said “prefrontal cortex” I was being a bit imprecise, using that term to describe the movement planning regions, which include the prefrontal cortex (as I understand it, the dominant player), but also the basal ganglia and the medial dorsal nucleus of the thalamus. I see these regions as receiving the experiences and feelings, but only generating them indirectly through their role as conductor of predictive simulations (i.e. imagination).

        I understand the claustrum / insular cortex region to be involved in generating the sense of self, or perhaps more accurately, modeling that sense based on information from the brainstem. So I could agree that they have a relationship to at least the interoceptive senses, although I’m not sure about the exteroceptive ones.

        I’m not sure about them generating feelings though, although I’ll admit I don’t know enough about them to discount that. My understanding is that emotional impulses start in the brainstem, although they’re often triggered by higher but still sub-cortical structures such as the amygdala, the cingulate cortex, and hypothalmus.

        But there are a lot of neuroscientists who insist that these lower levels are only involved in generating avoidance / attraction impulses, and that actual emotional feeling only arises in the neocortex, instigated by the sub-cortical processing. It feels like the actual answer is that emotional feelings arise in layers. Perhaps the dividing line is where we think we become conscious of the emotion, and I see it as a reasonable position to say that only happens in the neocortex. But I’m not set in that view.

        Okay, I’ll stop there and give you a chance to react and maybe present an alternate picture.

        Like

        1. Hi Mike, you really earned my respect as a person being so informed and taking time to respond to each comment carefully and thoughtfully. That’s a serious strength of this blog.
          That’s why I find it a little bit frustrating I don’t understand your position about consciousness. I really want to and above discussion was already a little bit beyond my comprehension so then don’t get involved you might say.
          But I think it’s true, and you as an informed person probably know better, that the feeling of pain comes after you have pulled back your hand from the stove. Actually it’s my understanding that the subjective conscious experience often comes after the initial movement. So how can conscious experience play a part in movement decision?
          You are actually saying that certain part of the information exchange between brain parts is consciously felt. Namely the information exchange that plays a role in movement decisions. That information flow is consciously felt and the conscious feeling also plays a role in the decision making. If it feels bad don’t do it. if it feels good moves toward it.
          Is this how you see it or is this too simplistically put?

          Liked by 2 people

          1. Thanks Oscar. So, I think the answer to your question is to consider a scenario from two perspectives.

            When I take a shower in my home bathroom, I can generally do so with little conscious thought. I’ve taken a shower in my own bathroom thousands of times, so it can all be done by habit. But if I’m staying at a hotel or friend’s house, where things are laid out differently, the faucet controls are different, the level of hot water, etc, I generally have to be more conscious about what is happening. In both cases, if I drop a bar of soap on my toe, I will react reflexively before consciously processing what happened.

            Okay, now let’s consider it from the movement planning perspective. When I take a shower in my home bathroom, I can generally do so with little movement planning. I’ve taken a shower in my own bathroom thousands of times, so it can all be done by conditioned mental reflex. But if I’m staying at a hotel or friend’s house, where things are laid out differently, the faucet controls are different, the level of hot water, etc, I generally have to do more movement planning of my actions. In both cases, if I drop a bar of soap on my toe, I will react reflexively before the movement processor can run a simulation on what happened to update models for later use.

            The movement planning regions, notably the prefrontal cortex, light up more in brain scans when someone is attempting to do something novel, and light up a lot less when they’re doing something routine. It doesn’t seem like a coincidence to me that this happens to match when we tend to be conscious about things.

            I should emphasize that this doesn’t mean I think consciousness lies entirely in the prefrontal cortex, but that it’s the conductor of a vast orchestra that takes place all over the brain. But, along with the rest of the frontal lobe, it does seem like a late stage integrator of conscious experience and feelings.

            Hope that makes things a little bit clearer.

            Liked by 1 person

          2. Mike,
            I think you might have missed the point of Oscar’s question here. I believe he was talking about conscious function, and specifically how pain after you screw up, can motivate you to not screw up given that it happens afterwards. There I think you need to discuss your concept of “simulation engine.” Or as for me, the conscious processor interprets inputs and constructs scenarios in order to figure out what to do. Thus simulating that you might burn your hand before you do, should explain what happens there.

            Liked by 1 person

        2. [thought I posted this already, but there may have been some trouble, thus the delay]

          Mike,

          [You probably have a better grasp on brain physiology than I, so what follows is likely to contain technical errors about specifics. I ask that you focus on the gist/”big picture” of what I’m trying to say]

          Damasio suggests that people have (at least?) three separable selves. I think he calls them the proto-self, the core self, and the autobiographical self. The first two are associated with the brain stem and [the midbrain?]. In any case, the autobiographical self is the one that most people are talking about in regards to consciousness and experience, and so that is the one I will focus on.

          My very speculative conjecture is based on my current understanding of a few physiological facts, about none of which I feel particularly secure in my belief.

          (Almost?) all of the sensory neurons pass through the thalamus and out to the neocortex. But approx. ten times as many neurons come back from the neorcortex to the thalamus.
          The entire neocortex can be considered a sheet made up of repeating units called “cortical columns”. Each column is a couple hundred neurons or so. The organizations of the columns differ to some extent in different areas, presumably to handle different kinds of signal processing. Cortical columns interact with neighboring columns, but each column (I think) has (at least?) one (pyramidal?) neuron that goes back to the thalamus.
          Different parts of the thalamus interact with all the other subcortical structures (basal ganglia, hippocampus, amygdala, etc.).
          Crick and Koch believed the claustrum to be involved in “orchestrating” consciousness, possibly involving synchronizing particular neurons at a given frequency (40hz?). Recent experiments show electrical stimulation of the claustrum can seem to turn off consciousness. Also, axons from claustrum neurons can reach the whole neocortex.

          So from these (possibly mistaken/incomplete/whatever) facts, here’s my wild conjecture:

          The neocortex is the screen in the Cartesian Theatre of the autobiographical self. Each axon that comes back from the neocortex to the thalamus is a semantically pointing pixel. All the heavy lifting, signal processing is done in the neocortex, but we are only (autobiographically) aware of the results that come back to the thalamus. The claustrum, I further wildly speculate, is the spotlight, which performs the function of generating attention by synchronizing a subset of those cortical axons.

          So, I speculate, the set of axons coming from the neocortex to the thalamus represent the manifest image, the umwelt, of the autobiographical self. An “experience” happens when one or more of those axons gets attended to by being synchronized to an appropriate frequency which results in some further processing which generates an output, such as creation of a memory via the hippocampus, or release of hormones via the amygdala, or something. The key is, there is only experience if there is output. No matter how many or which of those cortical axons “light up”, an experience only happens if something is done with them.

          I think this model can explain things like:
          – blind sight: the visual signals get processed, but something is disrupted in the path back to the thalamus or in the synchronization needed for attention and further processing.
          – delayed awareness of decision making: decision happens in neocortex, awareness requires firing up the attention engine after the fact
          -automatic behaviors: for some behaviors you don’t need the attention engine at all.
          -subliminal messages: maybe only the memory function needs time to rev up the attention engine. Maybe some signals generate output through the thalamus to the amygdala even when the attention engine is not engaged. (There might also be end-run signals that go straight from neocortex to amygdala.)

          How’s that?

          *
          [donning armor]

          Liked by 1 person

          1. James,
            Sorry. It looks like your first comment got eaten by the spam folder. If that ever happens again, just post a quick short comment to let me know, or drop me an email (About page) and I’ll fish it out.

            You know a lot more than most people about brain anatomy. Indeed, there’s so much to all this and it’s so complicated, and so much that I myself don’t yet know, that I’d be hesitant to accept that I necessarily know that much more than you about it.

            I completely agree that the role of the thalamus is critical. Indeed, the thalamus is the one structure that, if sufficiently damaged, erases consciousness completely. Damage to portions of the neocortex can knock out skills and various aspects of consciousness, but destroy the thalamus, and for all intents and purposes, the person disappears. Of course, destroy the upper brainstem, and you get pretty much the same result.

            When I talk about the prefrontal cortex doing this or the insular cortex doing that, in my mind I always have an asterisk by it that the functional module should include the corresponding thalamic nuclei that projects out to it. I don’t say it explicitly every time because it just gets tedious.

            The thalamus is often described as something of a network switch, passing signals from the senses to the cortices, and from the various cortices to each other, but based on what I’ve read, I think it’s more accurate to view it as something like the high level subroutines which farm out detail work to lower level subroutines in the cingulate cortex and neocortex.

            All of which is to say, I think the thalamus and the cortices form an integrated system, and talking about one of the other in isolation when it comes to functionality feels artificial to me. One piece of evidence (that you probably know about if you’ve read Damasio) are hydranencephalics, children born without a cerebral cortex, or much of anything above the thalamus. Having only the thalamus gives them a primal consciousness similar to newborns, but they remain profoundly disabled, unable to even move much.

            You’re right that all of the senses go through the thalamus, except one, smell. Smell has pretty much always (evolutionarily) had a weird pathway.

            And it would be wrong to say that the neocortex only talks to the rest of the brain through the thalamus. There are legions of independent connections between the neocortex and other structures such as the amygdala, hippocampus, basal ganglia, etc.

            I can’t say I’m much of a fan of Koch. He seems inclined to chase a lot of nutty ideas. He’s taken with Tononi’s IIT, which I think has some insights, but is ultimately incomplete, and with panpsychism, a view I find utterly unproductive. So I’m suspicious of his recent fascination with the claustrum. There does seem to be some consensus that this region is involved in sense of self, but it feels to me like Kock may be overstating its role.

            I’ve always seen brain wave frequencies as similar to the power refreshes that happen in computer circuits. It’s a basic piece of maintenance necessary to maintain arousal and processing at the proper level. But I’m open to being convinced otherwise.

            On the “Cartesian theater”, I’m a little uncomfortable with that phrase since it carries some baggage, but I’ll use it for this discussion. The amendment I’d make is that I think the audience is more in the frontal lobes (and associated thalamic and basal ganglia nuclei) than the neocortex overall. A substantial part of the back (parietal and occipital) and side parts (temporal) seem to be involved in generating the content for that theater, which in my mind includes sensory information, a sense of self, emotions, and any other aspect of experience you might care to name.

            This makes sense if you think about it, since it’s in the frontal lobes where the sensory information has to be assessed as a guide to action.

            For blindsight and all the rest, I think it pays to remember that sensory information also gets processed in the upper brainstem (superior and inferior colliculi), albeit at a much lower resolution. This processing appears to be below the level of consciousness, although we often feel its effects.

            So I see a lot of overlap between our views, sans the Koch stuff.

            See, you didn’t really need that armor after all 🙂

            Liked by 1 person

  14. Eric,
    I was actually talking about the function of consciousness. In my understanding, the movement planner (what is more often called the “executive center” in neuroscience literature, but I think “movement planner” helps us keep the primary evolutionary function in mind) is the conductor and primary nexus of the simulation engine (imagination), which actually depends on processed information from most of the neocortex.

    In other words, conducting simulations of various courses of actions and weighing the predicted consequences of each one, is how the movement planner goes about its task. The planning (simulations) may be about movement to be made in the next second, next hour, next day, next year, etc. Obviously, the more in the future, the more abstract the planning is.

    And the movement planner is constantly running simulations (imagining) to ensure that movement models are up to date for when the motor cortices need to actually initiate movement. Which is why the simulations may also be about past events (episodic memory) as well as possible future ones.

    Does that adequately explain how I understand the relationship between the movement planner (executive center) and the simulation engine (imagination)?

    Liked by 1 person

    1. Yes Mike it does explain it. I guess what threw me was when you were talking about the difference between the shower that you’re are familiar with and one that you aren’t. I guess your point was that you must run more simultaneous for the unfamiliar shower. Then the reaction to dropping the soap in either shower should be about the same, since it’s automatic, so perhaps you threw something extra in there. Whether from your “simulation engine,” or my “thought,” the hot stove is avoided given understood consequences of touching it.

      Some day someone should become quite a notable historical figure for finally nailing this stuff down. I’ll be interested to see if what ultimately becomes accepted diverges from my associated models, and especially since I connect them with models that go much further in other ways.

      Liked by 1 person

      1. There are a lot of people vying to be the Darwin of consciousness (Giulio Tononi is currently the most famous). But the more I learn about this stuff, the more I wonder if there ever will be, not because consciousness is unsolvable, but like the old concept of biological vitalism, we use the word “consciousness” to refer to too diffuse a collection of mechanisms.

        Each of those mechanisms may have their own Darwin like figure recognized within their particular sub-field, without any one dominating enough to be generally perceived as the person who “solved” consciousness.

        In contrast, in the book I’m currently reading on the prefrontal cortex, the author, Elkhonan Goldberg, mentions consciousness, quickly defines it as more or less what happens in the neocortex, and then announces he plans to stay away from the subject. This despite the fact that a lot of the mechanisms he’s discussing seem intimately connected with it. Many scientists still seem to see it as too vague a subject to bother with.

        Liked by 1 person

    2. Mike,
      If you know that consciousness is solvable, then it just doesn’t make sense for you to become more pessimistic as you supposedly “learn.” To me learning is more about fitting things together. Of course I know that you were just spittballing there, and so am I, but let’s try to stay positive and make some useful connections.

      To me what you’ve just said suggests the need for a person with the type of theory that I’ve developed. Can it really be useful for neuroscientists to be looking for “consciousness,” a term like all the rest that has no true definition, before we have useful definitions for the stuff? No, of course not. And yet that’s exactly what some are trying to do. Science needs to straighten out that joke that we call a Wikipedia consciousness page, for example, before we effectively document this stuff in brains. Otherwise we won’t even conceptually understand what we’re talking about. (So Tononi the panpsychist is actually the most famous consciousness theorist right now? Bloody hell!!!)

      Now that you mention it, I like how Elkhonan Goldberg gives consciousness a cursory “whatever” definition, and then goes on to play his own role, which I presume is to document brain dynamics in general. Perhaps he understands that it’s not yet time for the neuroscientist to truly contribute to our consciousness understandings. Perhaps he’d like to see some good architecture laid down rather than take such a vague leap.

      Liked by 1 person

      1. Eric,
        I think you might have missed my point. I wasn’t saying that consciousness can’t be solved, only that I’m not sure if there will ever be just one solution. “Solving consciousness” is starting to feel like “solving biological life”. There is no one quality that separates living processes from non-living ones, just a huge variety of different mechanisms. I’m increasingly suspecting consciousness may be similar terrain.

        When I first started this blog, I defined consciousness as more or less, intelligence + survival instincts. Obviously there’s a lot of complexity in both “intelligence” and “survival instincts”, but I’m increasingly coming back around to that view as a quick and dirty summation.

        Goldberg is actually far more scathing in his views of consciousness than I might have implied. He opined that it’s nothing but a code word for “soul”, and that the only reason we still talk about it is that “old gods die hard”.

        Liked by 1 person

    3. Mike,
      I agree that “life” is another such conundrum, though it doesn’t actually exist, does it? There is no “life,” there are simply more and less useful definitions for this humanly fabricated term. We’ll surely find some good definitions for it some day. Nevertheless biologists do seem quite able to explore biology without effective definitions for “life.” We’ll take it!

      Conversely I suspect that a highly effective definition for “consciousness” would have extremely practical uses within our softest sciences. The psychologist, psychiatrist, sociologist, cognitive scientist, and so on, seem amazingly hindered given that they do not yet have a basic theoretical comprehension of how we function. I mean to provide them with exactly this sort of understanding, and it contains no neurology. Once good “architecture” is finally nailed down, neurologists should then be able to use it to explore the “engineering” side of things.

      “He opined that it’s nothing but a code word for “soul”, and that the only reason we still talk about it is that “old gods die hard”.”

      I’m starting to really like this Elkhonan Goldberg! It sounds to me like he’s read the Wikipedia consciousness page. It sounds to me like he’s aware that a panpsychist happens to be our most famous consciousness theorist. And it sounds to me like he wants to do his job on the basis of modern understandings rather than funky conjecture. 🙂 But then I’m just speculating.

      Liked by 1 person

      1. Eric,
        You could say that life as a fundamental thing doesn’t exist, but I think you could say the same thing about consciousness. I don’t believe either of them are anything fundamental in the sense of electromagnetism, gravitation, or the nuclear forces.

        I see both being composite phenomena, higher level patterns, which means that in both cases, there are things that are more or less alive and things that are more or less conscious. Viruses are a good example of a borderline life case; people argue whether or not they really are alive. In the case of consciousness, there are plenty of examples of simpler animals who seem minimally aware, or humans who have suffered brain injuries or other impairments who fit the category “minimally conscious”.

        Goldberg is an interesting figure. He’s an immigrant from the old Soviet Union, a neuropsychologist and clinical neurologist. I picked up his book to get more info on the action oriented side of the brain, and I’m getting it, but I’m also getting a raft of his own pet theories. When reading that kind of stuff, I try to make sure I keep the author’s unique views separate from the general consensus of the field. Not that some of this theories aren’t interesting. He also intersperses neuroscience with biographical stories, mostly from his Soviet days.

        Liked by 1 person

        1. Mike,
          I commonly come back to my first principle of epistemology and, simply because I believe that it would help science enormously. You’ve never disputed it, but still tend to want to take shortcuts. This is great actually, since you thus provide examples which demonstrate how I’d like to improve science in this regard.

          What we call electromagnetism, gravitation, and the nuclear forces, are by definition no more real than any other aspects of reality. In truth these are simply terms that we use, and so can be arbitrarily defined in an assortment of ways. Nevertheless in extremely recent times we’ve developed quite “useful” definitions for these terms — nothing more.

          When we first began talking, I had you observe Newton’s definition of force as a product of accelerated mass. It’s not true, though quite useful. Notice that we also call gravity a kind of force, not because it reflects accelerated mass, but because it behaves very force like regardless. So here the definition of force has been changed for a different context. Thus “force” doesn’t actually exist, but rather is a term with more and less useful definitions.

          Conversely “consciousness” and “life” are very old terms, though neither have developed very useful definitions so far. Saying that viruses are borderline life is “is-ing” the term, like it exists out there. It doesn’t. Nevertheless a person might usefully define “life” such that viruses are included, or quite the opposite. The only important thing is that we use the provided definitions of a given theorist in our attempts to understand their points. Similarly I’d like to demonstrate the full nature of my “consciousness” definition for you and your readers to consider, because I suspect that it would be very helpful in general if science had such a definition at its disposal.

          Regarding Goldberg, yes he does sound like a character, though I was probably imagining him to suit my own purposes. He calls himself a neuropsychologist and clinical neurologist? Oh my! Those terms imply the opposite of what I was fantasizing. It’s as if I go to his office, he scans my head with various instruments, and then answers all sorts of personal questions about me! (Yes that’s ridiculous of course, but it’s also how those titles might reasonably be taken.)

          Liked by 1 person

          1. Eric,
            On definitions, have you considered that your definition of “real” might not be the most useful one you could be employing? I ask because my definition of it more or less matches what you describe, that is, for us to label some notion as “real”, that notion must be predictive to an adequate extent.

            I deem gravitation to be real because my theory of it adequately predicts what will happen if I let go of my phone a meter above the floor. I equally regard electromagnetism as real for similar reasons. (I have to take scientists’ word for the nuclear forces.)

            Or perhaps I should ask, what is your definition of “real”?

            A lot of this gets into which theory of truth we want to use.

            Anyway, on life and consciousness, my whole point was that the common understanding of these terms doesn’t have sharp boundaries, that the terms are somewhat amorphous. Redefining those terms and then solving for that new definition may be fine, but only if people see a strong resemblance to the common conception in your new definition.

            I think Goldberg’s version of neuropsychology is finding the correlates between neural states and psychological ones. I don’t see that title used that often. Most similar practitioners usually just identify as both a neuroscientist and a psychologist, or sometimes as a cognitive scientist, although it seems like that latter terms is just a new one for experimental psychologists.

            Liked by 1 person

        2. Mike,
          I personally like to have what’s “real,” “true,” or “exists” remain perfectly consistent with each other. But then notice how accepting I am when our friend JamesOfSeattle defines “reality” to include an element beyond that which “exists.” As I see it, it’s the obligation of any earnest pupil to do exactly that. (Furthermore I appreciate how he doesn’t consider languages such as math and English to “exist” beyond their human manifestations, and so puts them under a more broad classification of “reality.”) So yes, there may be definitions of reality that are more useful in general than my own.

          I do have a quibble with your “predictive” condition regarding the concept of “reality/truth/existence,” though it turns out to mostly be my own problem. The thing is that I can only assess things phenomenally, and I know quite well how unreal my perceptions happen to be. It’s thought that “colors” don’t exist beyond conscious experience, for example. Thus I’m not quite comfortable attributing absolute terms like “truth,” “existence,” and “knowledge” to perceived phenomena. Given this observation professor Massimo Pigliucci once suggested that I use the term “noumenal reality” as opposed to “phenomenal reality,” or at least when I’d like to emphasize an ultimate distinction. Apparently this comes from Kant. In practice however I must admit that I rarely have made this distinction. I like to use such terms absolutely, and then hedge them with softeners such as “apparently.” Perhaps I should stop doing that and get with the program?

          Anyway your point was that we each believe in causality, or natural rather than supernatural function as demonstrated by dropping a phone from a meter’s height. I will not fully dismiss René Descartes’ scenario that there is something like an evil demon fooling me, though that sort of thing does seem quite unlikely. I’m a foundationalist in that I noumenally “know” that I consciously process information (or “think”), and a coherentist from that point on. The model by which my coherentism functions is stated through my second principle of epistemology, or:

          There is only one process by which anything conscious, consciously figures anything out. It takes what it thinks it knows (evidence), and uses this to assess what it’s not so sure about (theory). As theory continues to remain consistent with evidence, it tends to become believed.

          Regarding life and consciousness, you’re certainly right that common understandings of these terms harbor no sharp distinctions. I have no stake in the term “life,” but appreciate your observation that my definition for consciousness will need to remain quite consistent with common understandings in order to potentially become popular. I’m actually quite sure that it does have such consistency however. The far greater obstacle that I perceive is the difficulty of demonstrating the usefulness of a model in forums (unlike yours) which seek to discredit outsiders. You seem to have developed a more welcoming cocoon which fosters and strengthens good ideas. Who knows what might emerge one day?

          I’m happy to hear that the titles “neuropsychologist” and “clinical neurologist” are not common. As an outsider, I sometimes say things that insiders take quite disrespectfully. This does not help my general situation, though holding back can sometimes be difficult.

          Liked by 1 person

          1. Eric,
            On phenomena and noumena, while I think it’s good to understand the difference, maintaining only a phenomenal attitude seems difficult, if not impossible. As Tina and I discussed, all observation is theory laden. Our brains map sensory information to conceptual categories at a pretty low subconscious level. We are pushed into the noumenal waters by our cognitive machinery. If we think we’re avoiding it, we’re most likely mistaken.

            For that reason, my attitude is it’s better to wear my noumenal theories on my sleeve. That way if I have a blind spot, someone may see it. Of course, this requires that I be open to revisiting those theories when challenged.

            Convincing people on the internet is tough. (Actually convincing people is tough in any venue.) In truth, it almost never happens, at least during the actual conversation. In the decades of having these types of conversations, I can count the number of times on one hand, and it was usually a case where the person’s beliefs weren’t very strong going in. More often I might hear from people months or years after a conversation that I laid seeds that eventually changed their mind, and then only if we left on amiable terms.

            On “neuropsychology”, I should emphasize that I don’t see it very often. For all I know, it might be extremely common in the relevant circles. A reader of popular astronomy articles might think that “light year” is used far more regularly in astronomy than “parsec”, when in actual astronomy papers, it’s the reverse.

            Liked by 1 person

        3. Mike,
          To make sure that we’re using our terms in the same way, I cannot have much of an understanding of noumenal reality since this goes beyond the images, smells, and so on that occur phenomenally. The noumenal would even reference a demon deceiver that I can’t know about, should such a thing exist. I’m not entirely sure, though we may have been crossed up with our terms. I don’t consider this to challenge what you and Tina have said, but our cognitive machinery should push us into “phenomenal” waters, since the noumenal isn’t something that we have direct access to.

          Furthermore I personally like using terms such as “real,” “exist,” and “know” in the absolute noumenal sense, and then softening them up with terms like “seems,” and “theoretically” where applicable. This gives me the ability to speak exactly without being incorrect. I simply hate asserting that something is “true” regarding reality itself, when there is the potential that it’s actually false by means of my own stupidity, or even by means of an evil demon deceiver. It merely takes a bit more diligence to speak exactly, and so this is what I try to do.

          Note however that I don’t mind saying that I do know one real aspect of noumenally true existence. It’s that I think. At the moment it simply isn’t possible for this to not be the case from my perspective. As I see it this is the only true foundation that the foundationalist will ever be able to develop. Even if it is circular, it’s still the one unique aspect of reality that I know to be true. “2+2=4” simply is not “real” in the sense that my own thought happens to be.

          I see two reasons that it’s hard to change the minds of others. One of them is that we’re all naturally subjective rather than objective, which is to say that our own experiences are all that we have access to. Then secondly, it seems to naturally feel much better to be acknowledged as right rather than wrong when matched up against the ideas of someone else. I’d say that you and I do a reasonable job moderating ourselves in these respects, and that movement has occurred in each direction. It’s like we’re two knives that are somehow able to sharpen each other. I’ve found it quite enjoyable.

          (Beyond the openness that you display in your posts to the possibility of being wrong, I recall you mentioning that you were actually a theist for much of your life. I presume that this change could not have happened without a great deal of objectivity. If you haven’t yet written a blog post to point me to regarding how this happened, perhaps you could think about one?)

          Even given the challenges associated with convincing others, my optimism comes from the apparent validity of my ideas, as well as the youth of science itself. I’ve mentioned before that I decided back in high school that we’re all selfish products of our environments, and I even went my own route in college so as to potentially not get programmed into what I considered to be failed approaches. Then about two decades later, which was also a few years ago, I decided that I had things pretty well figured out, and so started blogging heavily. Of course even if my ideas were as good as I thought they were, it would still be difficult for others to acknowledge this for the mentioned reasons. But notice that Newton’s seminal work was published only 330 years ago, and also the magnitude by which the institution of science has so quickly changed us. Looking back 500 years from now, would it seem odd that our mental and behavioral sciences carried perhaps a 350 year delay before they hardened up as well? My theory suggests a very good reason for such a lag (which I’ll surely discuss in your next post). If my ideas do happen to be as good as I’ve come to believe, vindication should not happen centuries from now, but rather within the next few decades. Regardless, this stuff is far too much fun for me to ever willingly stop!

          Beyond my thoughts for your next post, I wonder if you see the challenges of being objective about the same? Is this because we’re naturally subjective, and also because it feels far better to be acknowledged as the superior thinker? Perhaps you have something to add regarding the difficult but vitally important quest to effectively evaluate the ideas of others?

          Liked by 1 person

          1. Eric,
            “I cannot have much of an understanding of noumenal reality since this goes beyond the images, smells, and so on that occur phenomenally”

            If I look up from the laptop I’m typing this on, I consciously see a TV. Now, the idea that what I’m seeing is a TV is a theory about noumena. But I’m conscious of it as a TV before I’m conscious of its details, its color, shape, etc. Indeed, I only become conscious of those details if I choose to focus on them. This is because my pre-conscious visual system has already mapped it to a TV before it enters my conscious awareness.

            Even if I had never seen a TV before, I would first have been conscious of it as an object of some type (another theory on noumena), again before I would notice its sensory components. Although if it was a new type of object I’d probably be more inclined to access the sensory details.

            But even the most primitive sensory details I have access to has been heavily processed before they enter my consciousness. I don’t see my visual stream in its rawest form. My visual cortex isolates me from it, almost certainly because it wouldn’t be very informative. The early visual processing is necessary to construct the visual experience.

            That’s why I said that our cognitive circuitry pushes us into noumena waters. Now, with effort, we can pull ourselves back toward phenomena to some extent, but if you’ve ever looked at visual illusions, you’ll know that this often easier said then done.

            That’s why I think it’s better to accept that we have a worldview through which we filter things, and work to fine tune it, than to attempt perception of the world free of that worldview. Admittedly, this is a matter of philosophy on my part.

            On theism, I have to start with a tangent. I’ve been a sci-fi fan from an early age. As I got older, sci-fi inspired me to learn more about science. But this caused a problem with my enjoyment of most sci-fi since a lot of it wasn’t consistent with the science I was learning. For a long time, I made excuses in my head for things like why the Millennium Falcon moved the way it did instead of according to Newtonian physics. I did this excuse making thing for so long it had become a habit, a sci-fi excuse-habit. (I never had to do it as much for literary sci-fi.)

            At some point though, I realized what I was doing, decided it was wasted mental energy, and forced myself to just enjoy the movies and TV shows as light entertainment. (For what inspired this, see this post: https://selfawarepatterns.com/2016/08/08/dont-trust-your-emotions-they-will-betray-you/ ) My stopping the excuse-habit itself became a new anti-excuse-habit, a habit of forcing myself not to engage in intellectual excuse making whenever I reflexively started to do it.

            Not long afterward, in my late 30s by now, I decided to try going back to church. It was then that I realized that I also had an excuse-habit for religious beliefs, and the anti-excuse-habit kicked in. You can probably guess what happened from there.

            Being objective is hard. I don’t think we can ever accomplish it 100% (see above). The difficult thing is we can never know for sure that we’re accomplishing it or failing spectacularly. And even if we are accomplishing it, when we contradict the subjective views of others, they will assume that they’re the ones being objective and that we’re the subjective ones.

            I don’t know if you’ve ever read Dale Carnegie’s ‘How to Win Friends and Influence People”, but it’s the best book I know for tips on convincing people of anything. The hard part is being open to being convinced ourselves.

            Liked by 1 person

        4. Mike,
          I agree with what you’ve said about how you have an awareness of things before you consciously perceive them, though that’s not what I was talking about (as far as I can tell). That seems more about subliminal perceptions that come before consciousness. I was talking about conscious perceptions being a buffer that may or may not represent what actually exists. This is essentially in the theme of your recent post about consciousness being an “illusion.” Realities should be quite different from sensual perceptions, and there may not be any reality to what’s perceived at all (as with dreams or even a supernatural deceiver). So I think there was simply a mixup regarding how I was using the term “noumenal,” or what exists itself that cannot be perceived directly, if at all. For example I wasn’t talking about visual illusions versus not, but all senses being an illusion that may or may not have any link to noumenal reality. I fully agree that we have a world view that must be filtered and fine tuned, and given that it’s ultimately a false representation. I’d call this view “science” rather than “philosophy” however (though it’s also true that I believe that philosophy does need to become a science).

          On theism, apparently the preachers had it right all along — science fiction IS the devil’s handiwork! Well anyway it’s good to hear your story. As for me, I come from highly religious Midwestern people, but my parents moved out to California back in the hippy days to have their family. We’d go to church from time to time, where I hated being thrown into awkward Sunday School social situations, but didn’t think too much about eternal implications. At about the age of eight a good friend started scaring the shit out of me about how if you didn’t commit yourself to Jesus (or whatever), then you’d go to hell. I puzzled and bargained in deep thought about this for years. For the most part I decided that I’d commit myself to Him, though without actually going to church (since what kind of bullshit god would make you do that?). But at thirteen or so there must have been something like your realization with the science/science fiction divide that made me question the whole thing altogether. Here it hit me that there must not be a god at all! Thus all sorts of inconsistencies that I’d been puzzling over started to make sense. So then after weeks getting my case together I went up to my father for a serious debate. The last thing I ever suspected was for him to give me a big stupid grin and say “You’re right!”

          Liked by 1 person

  15. Interesting post, Mike. You’ve got me reflecting on the reduction of percepts here, although my musings have nothing to do with your subject, so far as I can tell, so I’ll keep them to my subjective self. 🙂 Also, I haven’t read all the comments here, so I apologize if I say something that’s already been said.

    I agree that the thought experiment is a bit loaded. It helps to illustrate the philosophical problem, so that might explain why it’s so popular. There’s a children’s book called The Giver, and if I recall correctly, no one can see color except the protagonist (and maybe a few others, I don’t remember.) Anyway.

    As for your question, why can’t subjective experience be explained physically? I think it can, and it obviously is, to some extent. Or rather, in a certain way. Science can ‘cover’ all of experience, generally, and in its manner, but it can’t account for all manifestations and meanings of experience. Some of these are better explained by poetry and the arts.

    I think people take issue with this extreme stance that the totally of my direct experience is not real. But that’s a pretty extreme position, and from our previous conversations, I don’t think that’s what you mean. If there were no experience of yellow, for example, there would be no basis for causal or correlative scientific exploration of that experience.

    “So then, what explains experience? As I’ve noted before, I think to have any hope of answering that question, we have to be willing to ask what experience actually is.”

    I think you’re right about this question. We need to know what experience actually is. But in answering the question with some particular theory on brain evolution is jumping directly into explaining experience through the scientific causal framework, and that precludes the question. This is why I admire Husserl’s phenomenology so much. The point in doing it is to find out what is being experienced through pure description. We try to avoid crossing over into the realm of theorizing, or if we do find that theory or concepts are involved, we look at those objectively and try to see whether those have imposed themselves or whether they pertain to some necessary aspect of the experience, without which the experience could not be what it is. It’s actually a radical empiricism, and, despite what some philosophers have done with the phenomenology, it’s not a merely ‘subjective’ realm. The point is to discover what lies in the totality of the experience, in other words, what lies in our experience of the whole world, and what’s necessarily involved in order to experience in such and such a way. Sorry about the wordiness here. I sense this paragraph must look like “experience” wallpaper.

    “It’s worth noting that just because these experiences can’t be subjectively reduced, it doesn’t mean that the neural correlates can’t be objectively reduced.”

    I don’t understand what reducibility of percepts has to do with this question. The more I think about it, the more it makes no sense. Not your fault, of course. I’ve heard this brought up countless times in regards to this problem. In fact, I think we’re saying close to the same thing. Sure, I can’t describe the color yellow to someone who hasn’t seen yellow, but a wine connoisseur might face a similar problem when conversing with a college student who just drinks wine to get drunk. Some might be better at detecting fine distinctions than others, and we probably also experience differently based on our ability to articulate our experiences. In other words, maybe yellow can be further reduced, if we were to create new words through various comparisons and learn new distinctions. Actually, you probably don’t need to go much further than the paint counter at Lowe’s to find some dazzling examples of yellow, and maybe these could be further refined. Perhaps the same could be true for the scent of a rose, etc., I don’t know. But none of this matters for the question we’re asking, so far as I can tell. There’s undoubtedly a threshold beyond which we can’t reduce some percept, and we might not even have a name for that percept yet. But what does that mean? As far as I can tell, nothing. As you say, these very tiny percepts would remain within a scientific framework. I think simply because they’re subsumed under other more general percepts that science directly deals with.

    By the way, experience as it’s purely understood may not be a matter of “adding up” percepts. That’s a theory, of course, but my first impulse is to say this isn’t how we experience. If anything, it’s the other way around. The whole comes first, and it takes practice and language to make divisions.

    But anyways, there shouldn’t be some quest to find the atomic percept that can’t be explained by science. Theoretically, science should someday be able to include such things, and maybe already does indirectly, in a general way. But that doesn’t mean that science can claim to have ALL knowledge of experience, since it doesn’t try to explain experience as it’s experienced. There’s no mystical escape route here. There’s only a distinction between what you’re calling physical causality and other kinds of knowing and accounting for, and I think the latter not only exists, but is necessary if we want to know the full story. (If it didn’t exist, I wouldn’t bother writing a novel.)

    Liked by 2 people

    1. Thanks Tina!

      “my musings have nothing to do with your subject, so far as I can tell, so I’ll keep them to my subjective self. :)”
      Maybe something you’ll blog about at some point? (He asks hopefully 🙂 )

      “but it can’t account for all manifestations and meanings of experience. Some of these are better explained by poetry and the arts.”
      Definitely. I’m not much for poetry, but one piece of advice I remember from one of my fiction writing books, was that art, to be successful, needs to provide an emotional experience of some kind for the audience. But we’re also often drawn to art for what it can teach us.

      “I think people take issue with this extreme stance that the totally of my direct experience is not real. But that’s a pretty extreme position, and from our previous conversations, I don’t think that’s what you mean. ”
      No, I think denying the existence of experience is unproductive. When I use the “illusion” word, I usually specify that I’m referring to the idea of experience as something separate and apart from the brain’s internal communication, from its information processing. But one of the reasons for this post is to hear possible reasons why that stance might be wrong.

      I grasp what Husserl is trying to accomplish. Try to obtain the purest description of experience without it being biased by theories about what’s happening.
      Unfortunately, I’m not sure if it’s actually possible to separate our perceptions from our theorizing about them. One thing I learned from both the history and the philosophy of science is that all observation is theory laden. You can’t really separate yourself from it.

      Attempting to do so may mask underlying assumptions you may not even realize are there. But describing those theories brings them into the light and enables others to point out possible issues with them. Of course, that means we have to be open to revising or discarding our theories. But this is one of the reasons I blog, is to put my thoughts out there and see what cases people can make against them.

      “The whole comes first, and it takes practice and language to make divisions.”
      I think this is astute observation. I’ve given some thought to this, and my answer is a bit complicated. There is the generation of the perception and the conscious receiving of that perception. I think our neural machinery constructs higher and higher level concepts from the initial primitives coming in. So the primitive divisions are generated first, then the more developed whole.

      But I agree that what gets consciously received first is the whole, because that’s more adaptive. I see the charging bear first, not the details of its anatomy. We have access to the more primitive components (down to a certain level) but only draw on them selectively. I agree that training may induce us to draw on some primitives we might never otherwise notice, but we can never take in all the details all the time; our brains don’t have the capacity for it.

      “But that doesn’t mean that science can claim to have ALL knowledge of experience, since it doesn’t try to explain experience as it’s experienced. ”
      Agreed. I actually think that’s one valid point that Mary’s room does make. Science can’t convey the raw experience of an observed system, no matter how much we know objectively about it. We’ll never know what it’s like to be a bat, or any other kind of non-human animal. Strictly speaking, we’ll never know what it’s like to be each other, although both being human, we can surmise that there’s a lot of overlap in our experience of the world. As you said, there’s nothing mystical here, just an epistemic limitation.

      Liked by 1 person

      1. As for my musings, believe it or not, I’ve forgotten them. So much for blogging about them. That might explain why I post so infrequently. I used to come up with ideas and if I didn’t have time to write the post, I’d start a draft and just write the title. That was a better strategy than relying on memory…in fact, the last post on Faith and the Good Thing was one of those subjects I’ve been meaning to write about for a long time—months now—but I kept forgetting to do it. Then I saw Charles Johnson’s name on the schedule for the book festival, and I decided it was time to get my act together.

        “One thing I learned from both the history and the philosophy of science is that all observation is theory laden. You can’t really separate yourself from it.”

        That’s critique of Husserl’s phenomenology that makes a lot of sense. As I understand it, he wants us to temporarily drop Kantian noumena, or any theory that relies on it or assumes it. Really, most of our typical ontological assumptions. (Our ideas—like, ‘book’, ‘river’, ‘equality’, etc.—remain as phenomena, since they are actually experienced.) But still, even if we take ‘theory’ in this sense, it’s very difficult to avoid accidentally sneaking it into our observations. I think he wanted a community of phenomenologists to check each other. His students went on to do this work, or so I’ve heard. And Heidegger doesn’t count (for me) as a continuation of this work, but that’s a topic for another day.

        On the whole coming first, that’s one I got from doing phenomenology. I’d like to take credit for it, but I might have read it in Husserl. Who knows. I think it came to me when I considered whether wine snobbery had any justification for those who don’t experience wine in that way. (A lot of those descriptions sound like BS. How do we know it’s not?) In the end, I decided it did. I realized that I’d learned to become more nuanced in many ways, and there was no reason to assume that others were not more nuanced in gastronomy than I could ever be (or want to be.)

        “There is the generation of the perception and the conscious receiving of that perception. I think our neural machinery constructs higher and higher level concepts from the initial primitives coming in.”

        Funny how the two seem to move in opposite directions. How does our neural machinery construct higher concepts from primitives? (I’m assuming ‘primitives’ means something like an atomic, not-quite-yet-experienced percept?) Does this have to do with thresholds of perception? For instance, heat might not register as heat until it reaches a certain degree?

        “I agree that training may induce us to draw on some primitives we might never otherwise notice, but we can never take in all the details all the time; our brains don’t have the capacity for it.”

        There’s one place where phenomenology and science coincide. This directed-ness towards objects, and necessary exclusion of others (what I call the background) is called intentionality. Except in phenomenology, there wouldn’t be a brain explanation. No explanation of why or how we have it, actually. Just the description of intentionality, noting how pervasive it is, perhaps all-pervasive, and perhaps fundamental to the way we experience.

        Liked by 1 person

        1. “I used to come up with ideas and if I didn’t have time to write the post, I’d start a draft and just write the title.”
          I used to do that too. Now that I think of it, it was a pretty good strategy to keep ideas bubbling and posting frequency higher. Hmmm. Might need to get back into that habit. It does leave a lot of “zombie” draft entries lying around though.

          “I realized that I’d learned to become more nuanced in many ways, and there was no reason to assume that others were not more nuanced in gastronomy than I could ever be (or want to be.)”
          I think the beginning of wisdom is coming to understand that there are complexities to things we don’t understand or have less familiarity with. (As a manager, I learned long ago that everybody’s job is more complicated than it looks from a distance.) I know mechanics who can tell by the sound of a car motor what make and model it is, and how well it’s working. I used to be able to tell when a hard drive was approaching failure by how it sounded. Our ability to make out details in sensory data becomes amazingly discriminatory with enough repeated exposure.

          “How does our neural machinery construct higher concepts from primitives?”
          A lot of it is recognition of previously understood object types. Have you noticed that children tend to stare at things far longer than adults? Or that they enjoy watching the same movies over and over again? I think it’s because they’re still building recognition patterns. An adult sees those things, maps them to preexisting patterns, and is ready to move on. (The problem, or course, is that it can cause us to fail to recognize a completely new pattern that is outside of our previous experience.)

          What’s interesting about this is we appear to be pre-wired to recognize certain types of patterns, such as faces, or social arrangements. Recognizing things far outside of our original ecological niche takes a lot more work.

          For me, intentionality seems like another word to describe these recognition patterns, these models that we build of objects in environment. Understanding exactly how that works seems like a major goal of cognitive science and artificial intelligence research, although the AI stuff gets better every year.

          Liked by 1 person

          1. Speaking of zombie drafts, I think that’s why I ultimately stopped doing that. Here’s the stupid thing—I had multiple drafts of the exact same subject. I’d start a new draft, forgetting I had that old one. It’s like keeping a planner and failing to look at it, thereby missing an appointment. (Timidly raising my hand here.)

            I still have my very first computer from my sophomore year of college and I got it started a few years back for just long enough to email myself a bunch of old papers and my undergrad thesis. I still had one of those clunky wifi cards, believe it or not. That was the only time I’ve ever heard a hard drive fail. Actually, I’m not sure if you’d call it a fail if it just turns off?

            As for constructing concepts from primitives, it sounds like “patterns” are higher concepts than, say, “cup” or “dog”? I wonder if causality would count as a pre-wired pattern? And since certain patterns are pre-wired, do we combine these patterns with experience to create both lower concepts and new patterns?

            Speaking of social patterns, I’d love to find out what it is that makes Geordie decide whether he likes another dog or not. I keep coming up with theories, and he keeps making me revise those theories. There seems to be some rules we can count on when it comes to dog behavior, but it all happens so fast.

            On intentionality, I may have been confusing in describing it. I think it’s broader than recognizing patterns, although that would be included. All it means is attending to some object, or thought. We find that we always are (so long as we’re either dreaming or conscious) and in doing so, we find that there’s always something not being attended to. This might be called a pattern, I don’t know. Or a general rule about how we experience, since we can’t experience something without the rest of the world hanging out in the background.

            On AI, have you seen “Spy in the Wild”? It’s been on PBS (BBC) a few times. It’s about animal robots used as hidden cameras, and the footage they capture is pretty amazing. The robots even fooled me a few times. They’re pretty amazing. Here’s a blurb on it:

            http://www.telegraph.co.uk/news/2016/12/31/bbc-robot-animals-go-undercover-film-nature-even-david-attenborough/

            Liked by 1 person

    2. On zombie drafts, I know what you mean. I still have 50 floating around in my drafts tab. (Just checked.)

      My first computer was an Atari 400 which got sold in a garage sale long ago. Life was much simpler when I had that computer.

      My use of the word “patterns” pretty much denotes anything above the raw sensory information. In reality, from what I’ve read, we don’t have access to the very raw information. If we did, for vision we’d perceive two separate streams, each having a narrow spot in the center missing, surrounded by high resolution imagery near the center, but with the resolution rapidly fading toward the edges, and a constantly shifting field of vision as our eyes move involuntarily. Our unified and stable field of vision is a creation.

      So dogs and cups would definitely be patterns, albeit lower level ones than, say, the concept of the Kennel club. In fact, a dog would be a multiple modality pattern, having visual, touch, smell, and sound aspects. I read somewhere that dogs always light up the same part of the brain, indicating that they may be one of those concepts we’re wired for, probably originally used for wolf recognition.

      On Geordie’s evaluation of other dogs, smell is probably a big factor. Remember that he has a much richer experience of it. There are probably things dogs can tell about each other from smell that we have no awareness of. Just as there are probably things about humans you can see that Geordie probably can’t perceive at all.

      On intentionality, I think in order to attend to something, we first must recognize it at least as an object of some kind, (think of the illusion of water far ahead on the road), and then recognize it as a specific object. This all seems to happen below consciousness unless it’s a type of object we’ve never seen before. Usually we’re just conscious of the object itself.

      I think I’d heard of those animal robot spies, but I haven’t seen the show. Thanks! I’ll check it out.

      Like

  16. [doffing armor … whew … now where is that asbestos Darwin of Consciousness hat?]

    [ran out of reply space on previous chain … so …]

    Mike,

    I appreciate that areas of the neocortex can interact with basal ganglia, etc., without going thru the thalamus, but my point was those interactions would be subconscious relative to the autobiographical self.

    As for Koch’s interest in the claustrum being recent, see Crick and Koch’s article from 2005: https://www.ncbi.nlm.nih.gov/pmc/articles/PMC1569501/. Also, as is frequently the case, when I propose an idea that is significant, it usually means that someone has been working on it for two or three years already. In this case, it seems I am not the first to propose a connection between the claustrum and attention. See http://www.cell.com/trends/neurosciences/pdf/S0166-2236(15)00120-4.pdf. Also, You mentioned a connection between the claustrum and the self, which I have not heard about. Got any ref’s?

    As for Koch and IIT and panpsychism, I’m with you, … … except … [ah, there’s that hat] … I think I can explain how panpsychism isn’t completely wrong, and I think I can explain what’s missing from IIT. The second of these we’ve already started talking about, namely the semantic definition of information. The first part is a pretty long story that starts with matter and ends up at the thalamus/claustrum/etc. complex. It describes conscious agents as being organizations of conscious agents. So, your prefrontal cortex/thalamic nucleus combo could easily be described as a separate conscious agent. In fact, every neuron is a separate conscious agent, except it doesn’t have the capabilities associated with a high level conscious agent, like a whole brain. But I am going to say that the highest level agent below that of the whole brain is the thalamus/claustrum/basal ganglia/etc. complex, and that that agent is what is referred to as the autobiographical self.

    So, do you want the long version here?

    *

    Liked by 1 person

    1. James,
      On Koch and when he became interested in the claustrum, I stand corrected. Obviously he’s been interested in it for a while.

      On reference for the claustrum, I might have to make a correction here too. I thought I had gotten that idea from Damasio, but searching through ‘Self Comes to Mind’, I’m finding more references to the insula as part of the protoself. It seems like I read somewhere that the claustrum, which is right next to the insula, was thought to participate in that sense, but a cursory search isn’t pulling it up. I might be mistaken on this.

      Interestingly, Damasio assesses the proposed role of the claustrum in awareness, but thinks it’s too small to have the necessary computational substrate. He focuses on a region he calls the PMCs (posteromedial cortices) which is the middle portions of the parietal lobe, including the retrosplenial cortex, the posterior cingulate cortex underneath, and the precuneus underneath that. It’s worth noting that that all of these regions are in the same neighborhood and heavily interconnected with the claustrum and insula, so the answer might be more “all of the above”.

      I don’t know if you saw Tina’s comment, but I think we’re talking about the regions that construct the high level perceptions, the heavily integrated ones. But it’s built on lower level primitives worked out in other areas, many of which are themselves available to consciousness when it chooses to focus on them, at least down to a point.

      All of this, I think, is about the generation of the perceptions, not their consumption. For that, I think we have to move to the action oriented side of the brain in the frontal lobes. I think what these regions get is the distilled “prepared” versions rather than the gory details, which is one reason why individual perceptual primitives are not subjectively reducible.

      Anytime panpsychism comes up, particularly when we start talking about conscious neurons, my first question is, what definition of consciousness are you using? And why should we use that definition?

      I’m suspicious of any definition that doesn’t include awareness, emotions, and volition, that is predictive modeling of the sensory environment, assessing those predictions in terms of the system’s goals, and being able to simulate possible responses before actually acting. And many people’s intuition of consciousness won’t be triggered if the goals aren’t primarily biological, such as homeostasis or reproduction.

      Like

      1. Mike,

        I accept that the list of features you describe are pretty standard for human level consciousness, but I wanted to distill down to the “essence” of consciousness.

        So start with an agent that everyone agrees is conscious as having everything in the list you provided: awareness, emotions, volition, simulation, prediction. Now remove exactly one of those things. Is the agent still conscious? I would say that all of those things in your list require consciousness, but none of them are necessary for it. Instead, your list describes the things humans can do because of consciousness. Some humans can do some of the things, but maybe not all of the things.

        So my question became, what is the basis of consciousness? At first, my answer was information processing. But after some thought I decided I could take it a step further back and make the answer “interaction with the environment”. And so the model of the basic event is super simple:

        input –[agent]–> output

        where
        — input is a discrete set of physically measurable variables,
        — output is a different set of such variables such that the output would be different if the input were different, and
        — the agent remains essentially unchanged and capable of repeating the process.

        For reasons I won’t go into, I named this event a psychule.

        Now at the very bottom level, every physical thing can be plugged into the “agent” part. Thus panpsychism. It was either Socrates or Aristotle who defined existence as that which has causal power, i.e., that which can be an agent.

        So at this bottom level, given inputs and simple agents (say, electrons?) outputs can be predicted by known physics, and these kinds of events are not especially interesting in terms of consciousness. But by the nature of the definition, agents can be combined by connecting outputs of one agent to inputs of another agent, thus creating superagents, which we’ll just call agents, with the understanding that any given agent may be a combination of sub agents.

        Interesting things start happening when you combine agents. We would call these interesting things emergent because you would not be able to predict them [practically] from the parts.

        Probably the first interesting thing to emerge is semantics. In this sense semantics/meaning/value/purpose emerges when life emerges. Only when you have a reproducing thing do you have something that “cares” about information in the environment, thus making that information “semantic”. [side note: you can have things that are not reproducing that “care” about the environment. You just won’t see any of those things until something that is alive creates them.] The first agent to emerge at this level might be a cell-surface receptor.

        Probably the next interesting thing to emerge is communication. At some point it becomes valuable for the response (output) to an input to happen at a distance from where the input was recognized. Thus, if you combine one cell-surface receptor that can trigger another agent, which triggers other agents, you get an agent that recognizes an input on the outside of the cell and generates its response, say, deep inside the cell.

        The next interesting thing emerged with multicellular life. Communication is still important, but now you may want a response to happen many cells away from the recognition event. From the previous level you can have a cell with a cell-surface receptor that triggers another cell surface agent, which triggers more of the same, so that triggering one side of the cell can generate a response on the other side of the cell. Combining such cells together allows you to describe an agent that produces a response at essentially any distance. Thus, the neuron, Nature’s multipurpose agent.

        Obviously these agents (neurons) can be combined in serial as well as in parallel, and so we get all of the information processing capabilities these combinations can produce. At this point you can choose which information processing capabilities you require to count as “consciousness”.

        So finally we get the most sophisticated agent extant today [speculation much?]:

        Input (from corticothalamic axons) — [agent = thalamus,basal ganglia, etc.] –> Output (e.g., memory, increase hormone levels, creation of new concepts?, etc.)

        Note: the “agent” would not necessarily be described as conscious. The agent, as Dennett would say, is competent without comprehension. The agent produces consciousness without necessarily “having” consciousness.

        *
        [looking for asbestos suit to go with the hat]

        Liked by 1 person

        1. James,
          I very much appreciate the walkthrough. I can tell you’ve given this a lot of thought. You did an excellent job of describing what I perceive as something like the standard panpsychist framework.

          First, let’s consider if removing the qualities I listed leaves us with a conscious entity. Volition (action planning simulations) is built on awareness (sensory modeling) and emotion (assessment in terms of goals). You can’t have volition without the others. (Evolutionarily I think any of the three are unlikely without the others, but that’s a different topic.)

          Removing volition leaves us with a pretty robotic system. Humans who had a severe lobotomy might be an example. These poor people were still alive, but it seems like their personality had been largely extinguished. Were they conscious? Well they reportedly had sleep / wake cycles, so they were aroused, “conscious” in the sense of being awake, but the severe cases had the personality of an infant or “of an oyster”. It’s fair to say they had lost the essence of who they had been before. https://en.wikipedia.org/wiki/Lobotomy

          Finding healthy animals without volition brings us down to pretty simple creatures. Sea slugs and amphixious come to mind. These creatures don’t pass any of the standard behavioral tests usually used to measure some form of consciousness. (Operant learning, behavioral trade-offs, self-delivery of analgesics, etc.) Of course, these creatures also have limited, if any, awareness.

          We can remove emotion, but the result is a system that feels nothing and has no preferences, that really does nothing more than accumulate data. We can remove awareness, but it seems like what we’re left with is the agent concept you described.

          But here’s my problem. Why should we regard the information processing entities you described as conscious?

          Certainly conscious systems have to reduce to such entities, but then, so does a video game. Yet without sufficient features, no one would be tempted to regard any of these information processing pieces in and of themselves as a game. It would need to have some user interface, set of rules, and other minimal qualities before anyone would be tempted to regard it as a game. (The same would apply to any information processing application such as a phone, e-reader, etc.)

          Why should we be any less demanding with consciousness? I think consciousness is definitely information processing, but not just any information processing. What we call “consciousness” is a suite of information processing applications, a suite that I think most people need to see include the functionality I noted above in order to have their intuition of a fellow consciousness triggered.

          Of course, what we define to be consciousness is a philosophical decision. We could define it as any information processing system. But then I think the interesting distinction would simply shift from the difference between conscious and unconscious systems, to the difference between human / animal consciousness and that of protons, storms, and thermostats.

          Like

  17. The most profound Buddhist teachings talk about there being only Mind. What do they really mean by that? They are not talking about an object, a thing, with shape. It is not a source, not an absolute or substratum. It is a spontaneous generation of phenomenon which have no inherent existence apart from this generation of appearance. Mind and appearances are of the same substance. Your own nature and the appearances of the world are the same process. Once you begin to see the impossibility of separating anything from yourself, the search for answers to all of these conceptual questions begin to lose their steam. You discover a deeper truth. This truth is the nature of Mind, your own nature. To discover your own nature is to be at the center of your self. To be at the center of yourself is only a figure of speech because there is no center and no self, there is just this understanding or knowing that is present. In fact, there is no understanding and knowing about a thing. Whatever experience happens, there is no attachment or owning it. It’s there and it’s not there. Your own story is included and freed from your copyright!

    If you have a predilection for scientific work or investigation, perhaps approaching this from the above view might bring a very different experience into your lives. There have been individuals who have pointed this out for thousands of years, in all cultures, and so few take advantage of the ‘wisdom of the ages’ just staring us in the face. The elitism of the intellectual and its demand for proof is a kind of madness, a dis-ease. You are groping in territory that you are self creating, just like the dog chasing its tail. You think you are getting closer. Better to wake up from this dream. Your natural state doesn’t want it or need it. You all need a good cleanse! An ultimate enema of sorts. 🙂 No offense meant and I hope you take this in a spirit of openness.

    Liked by 1 person

    1. Jeff, understood on no offense. None taken by me. I hope you’ll accept my response in the same spirit.

      It seems like your first paragraph is describing what in philosophical circles is usually called “idealism”, the idea that the only things that exist are minds, and that the physical world is an illusion. My take on that kind of thing is that even if it all is an illusion, ignoring it appears to come with painful consequences. It seems like we have little choice but to play the game.

      On the ‘wisdom of the ages’, I have no real issue with people finding comfort and meaning in these types of beliefs. And Buddhism in particular seems to have contributed excellent mental strategies for coping with things we can’t change.

      But those beliefs and strategies are no substitute for scientific and philosophical investigation. I’m 50. If I’d been born more than 500 years ago, I’d almost certainly be dead by now, after having lived a short brutish life. What diseases did the ‘wisdom of the ages’ ever cure? What improvements in living standards or governance? Other than giving us coping mechanisms (admittedly no small thing), what has it done to elevate the human condition?

      Science is far from perfect, and it certainly doesn’t have all the answers. But if we’re looking for reliable knowledge, knowledge we can use to actually make a difference, it’s the best tool we have.

      Like

      1. Mike,

        This was not my intention to put forth any ‘thing’ that exists, including mind or minds. In fact, I thought I tried to explain this. Similarly, for things said to be illusory as opposed to ‘real’. All these dualities are undermined through the principle of the nature of mind which is knowing and empty of self and all divisive beliefs. The game is then played without a player, spontaneously happening without an owner. The thing that many don’t comprehend is that what I am talking about is experienceable and is not a belief in something like a concept. It is an experience of what is when your ordinary mind is woken up to its nature. It is no different than seeing ‘yellow’. You know that you are not seeing blue. There is no doubt in your mind about this. Just as you know that you are not me. You don’t need to think about it.

        No offense, Mike, but your questions border on naivete. Death is part of life. Whether it happens when you are 30 or 50, is not of ultimate importance. To live a life with wisdom, where the everyday neurosis is not present eating away at us seems quite important to me. This dis-ease is at the heart of every disease that science is trying to cure. It doesn’t take a genius to see that stress and obsessive behavior leads to all sorts of bodily problems. And, so far, science hasn’t made a dent in that. The human condition is a nightmare for a great majority of people on this planet presently. Do you really think science is going to solve that? Look at who is controlling most of the science. Look at the bloody prices of health care in the USA and tell me how compassionate all men/women of science are and how available all these drugs and procedures are to the common man. I think you need to reexamine the realities of this. It is primarily controlled by big business and they are not in the business of public service. I think we can agree on that, don’t you think? Science doesn’t have any of the answers to the real questions of life, Mike. Isn’t it apparent by now?

        I am not suggesting an abandonment of science, Mike. Far from it. It is just that you want something from it that you cannot get. It is not possible for science to give you what you actually need to live life to its fullest, moment by moment, which is the only life I know, not idealism. For whatever reason, somehow you’ve abandoned the more intelligent forms of philosophical enquiry that appear in Buddhist literature as well as in other forms of self enquiry in other disciplines. Probably, because you’ve equated the ‘message’ with the outward forms of many religious organizations with a kind of antiquated belief system such as the world was created in 6 days by god and he rested on the 7th day. These kinds of popular beliefs should be abandoned, but those beliefs are not what I’m remotely talking about. You need to have the conviction of direct experience of what you really are and that takes a supreme effort in the sense of commitment to real contemplation of the analytical, discursive thought process and the whole storehouse of knowledge/information that we call ME. Letting all of this go is the only way to refresh oneself and to re-cognize oneself.

        Like

  18. Hi Mike, this paper was highlighted in a Nature Reviews Neuroscience Alert I received recently:

    ‘Disrupting the experience of control in the human brain: pre-supplementary motor area contributes to the sense of agency’, Moore et al 2010
    http://rspb.royalsocietypublishing.org/content/early/2010/04/07/rspb.2010.0404

    … that I thought you might find of interest.

    Consciousness/experience does seem something like riding the crest of a wave of simultaneous automatic nonconscious evaluations resulting in affective valuations that coalesce into answers to the constant, ever-present, and ultimate BIG question: “approach or avoid?”, ie movement.

    “… all animate life, from the single-cell amoeba upward, is equipped with at least some basic form of approach-avoidance mechanism that produces or regulates movement toward potentially beneficial stimuli and away from potentially harmful stimuli.” – pages 5/6

    ‘Handbook of Approach and Avoidance Motivation’, Elliot(editor) 2008

    Looking at the table of contents I see Panksepp is a contributor, also LeDoux, Bargh, and Baumeister. I’ve wanted to look into the subject but haven’t been able to get to it yet.

    Liked by 1 person

    1. Hi Mark,
      Thanks for sharing the paper. You must have some interesting alert monitoring running.

      I have to admit that “sense of agency” hasn’t been on my radar with these investigations. But I can see how disrupting the premotor cortex would have an effect on it. Indeed, if you sufficiently mess with it, any resulting movement might actually be said not to have resulted from any type of agency.

      It doesn’t surprise me that Panksepp would be a contributor to a book like that. Wow, they want some serious money for it. I feel sorry for the students who have to buy it.

      I started reading Panksepp’s ‘The Archaeology of the Mind’, which is far more affordable, but ended up taking a break from it to start Dennett’s new book, then ended up taking a break from it to read Elkhonnan Goldberg’s book, ‘The New Executive Brain’ on the frontal lobes. I may get back to Panksepp eventually, but I found his attitude a bit off putting.

      Goldberg’s views seem to be compatible with mine, that subjective experience is generated by the back of the brain, but the audience for it is in the front. And imagination requires the front as a conductor.

      Anyway, good hearing from you!

      Liked by 1 person

Your thoughts?

This site uses Akismet to reduce spam. Learn how your comment data is processed.